You are on page 1of 109

Republic of the Philippines

OCCIDENTAL MINDORO STATE COLLEGE


Labangan, San Jose, Occidental Mindoro
website: www.omsc.edu.ph email address: omsc_9747@yahoo.com
Tele/Fax: (043) 457-0231 CERTIFIED TO ISO 9001:2015
CERT. NO.: 50500643 QM15

Learning Module
in
Logic

Compiled by:
PAUL B. CALARANAN

Academic Year:
2020-2021

The compiler does not own any of the contents of this learning module. Due credits and
acknowledgment are given to the authors, internet sources, and researchers listed on the
reference page. Such sources are reserved to further explain concepts and cannot be credited to
the compiler and the school. All diagrams, charts, and images are used for educational purposes
only. The sole objective of this instructional material is to facilitate independent learning and
not for monetary gains because this is NOT FOR SALE.

2020 Edition

i
Republic of the Philippines
OCCIDENTAL MINDORO STATE COLLEGE
Labangan, San Jose, Occidental Mindoro
website: www.omsc.edu.ph email address: omsc_9747@yahoo.com
Tele/Fax: (043) 457-0231 CERTIFIED TO ISO 9001:2015
CERT. NO.: 50500643 QM15

APPROVAL SHEET

This Instructional Material entitled LEARNING MODULE IN LOGIC,


compiled by PAUL B. CALARANAN (A.Y. 2020-2021), is recommended for
production and utilization by the students and faculty members of the Occidental
Mindoro State College.

PANEL OF EVALUATORS

Local Evaluation Committee

College of Criminal Justice Education

ANTONIO P. PERALTA Jr., MSCA MARIANY AILA MORALES, MSCJ


Member Member

KHARINE M. REYNO, PhD


Chairperson

Overall Instructional Materials Development Committee

VENESSA S. CASANOVA, PhD MA. IMELDA C. RAYTON, MA


Member Member

Recommending Approval:

JESSIE S. BAROLO, JR., MAEd


Chairperson

Approved:

ELBERT C. EDANIOL, EdD


Vice President for Academic Affairs

i
TABLE OF CONTENTS

Lesson 1: Introduction 1
Logic 6
Assessment

Lesson 2: Language 9
Assessment 11

Lesson 3: Diagrams and Analysis 14


Diagraming Arguments 15
Assessment 16

Lesson 4: Fallacies 18
Fallacies Based Emotional Appeal, 19
Fallacies of Unwarranted Assumption 20
Assessment 22

Lesson 5: Categorical Propositions 26


Quantity, Quality, and Distribution 27
The Modern Square of Opposition 28
Traditional Square 29
Assessment 33

Lesson 6: Syllogism 38
Standard-Form Categorical Syllogism 39
Diagramming in the Traditional Interpretation 40
Mood and Figure 41
Sorites 43
Assessment 44

Lesson 7: Propositional Logic 47


Compound Statements 48
Truth Tables for Propositions 49
Contingent and Noncontingent Statements 50
Indirect Truth Tables 51
Assessment 53

Lesson 8: Natural Deduction 57


Implication Rules 58
Replacement Rules 60
Assessment 62

Lesson 9: Predicate Logic 65


Translating Ordinary Language 67

ii
Four New Rules of Inference 68
Change of Quantifier 69
Demonstrating invalidity 70
Relational Predicates 71
Assessment 73

Lesson 10: Analogical Reasoning 75


Framework in Analysing Arguments 76
Assessment 78

Lesson 11: Legal Arguments 80


Assessment 83

Lesson 12: Moral Arguments 85


Value Judgments 86
The Naturalistic Fallacy 87
Assessment 88

Lesson 13: Statistical Arguments and Probability 90


Samples and Populations 91
Standard Deviation 92
Assessment 93

Lesson 14: Causality and Scientific Arguments 95


Causality 96
Mill’s Methods 97
Hypothesis Testing 98
Assessment 99

Lesson 15: Analyzing Essay 102


Assessment 103

iii
LESSON I
INTRODUCTION

TOPICS
1. Philosophy
2. Logic

LEARNING OUTCOMES
At the end of the lesson, student should be able to:
 Clarify and compare the appropriateness of accepted definition and the
role of Logic as an introduction for any philosophical inquiry.
 Indicate and present the relevance of critical philosophical thinking, clarify
the limitations therein.
 Orient students to the formation of ideas and the different possibilities of
philosophical errors of terms.

TOPIC 1: PHILOSOPHY

Philosophy, a discipline or an area of study? This causes endless problems when the
word "philosophy" is mentioned to laymen. Philosophy from the Greek word
'philosophy' meaning 'love of wisdom' - is referred to as the study of general and
fundamental questions of nature, understanding, beliefs, reason, mind and language.
These issues are also raised as concerns that need to be examined or answered.
Pythagoras (c. 570-495 BCE) possibly invented the term. A 6th-century BCE Greek
philosopher and mathematician, originally from Samos (an island off the coast of Asia
Minor settled by the Greeks), He resided in southern Italy, controlling city politics until
the turn of the century when the people revolted against his control and forced him to
settle instead in Metapontum. He was venerated by followers, who established some
kind of quasi-religious order. While he did not express his theories in written form,
during the 5th century BCE, Pythagorean centres sprung up across the Greek mainland,
including in Thebes and Athens, and he undoubtedly inspired Socrates and thus Plato.

Philosophy historically encompassed all bodies of information. Since Ancient Greek


philosopher Aristotle 's time to the 19th century, "natural philosophy" included
astronomy, medicine, and physics. Newton's 1687 Mathematical Principles of Natural
Philosophy, for example, later became known as a book of physics.

The rise of modern research universities in the 19th century contributed to the
professionalization and specialization in academic philosophy and other disciplines.
Some work that was historically part of philosophy in the modern period became
independent academic fields, including psychology, sociology, linguistics and economics.

Philosophy, in a broad sense, is a practice that people pursue in trying to understand


fundamental truths about themselves, the world in which they live, and their
relationships to the universe and to each other. The philosophy is almost the same as
that of an academic discipline. Many who study philosophy are actively engaged in

1
questioning, answering and arguing for their responses to the most basic questions of
life. Traditionally to make such a practice more formal academic philosophy is divided
into major fields of study such as Metaphysics, Epistemology, Ethics, History of
Philosophy, and Logic

Logic. The significant element of philosophy research is the explanations or reasons


provided for the answers given to these questions by the people. Philosophers employ
logic for this purpose to research the essence and structure of claims. Logicians ask
questions, such as:
 What constitutes "good" or "bad" reasoning?
 How do we determine whether a given piece of reasoning is good or bad?

TOPIC 2: LOGIC

Logic is the study of reasoning. Logic examines the level of correctness of the reasoning
found in the arguments. The argument is a group of statements, one of which (the
conclusion) is claimed to follow from the other (the premises). A statement is a
sentence which is either true or false. Every statement is either true or false; these two
possibilities are called "truth values." Premises are statements that contain information
intended to provide support or reasons to believe that a conclusion has been reached.
The conclusion is the statement which is claimed to follow from the premises. In order
to help us recognize arguments, we rely on the premise of the words and phrases
indicator and the conclusion of the words and phrases indicator.

Inference is the term used by logicians to refer to the reasoning process expressed by
the argument. If a passage expresses a reasoning process — that the conclusion is based
on the premises — then we say that it makes an inferential claim. If a passage does not
express a reasoning process (explicit or implicit), it does not make an inferential claim (it
is a non-inferential passage). The explanation is one type of non-inferential passage. An
explanation of why or how an event occurred is given. Explanations are not arguments
on their own; they can, however, form part of the argument.

There are two types of arguments: deductive and inductive. A deductive argument is
one in which it is claimed that the conclusion is necessarily based on the premises. In
other words, it is claimed that, on the assumption that the premises are true, it is
impossible for the conclusion to be false. An inductive argument is one in which it is
argued that the premises make the conclusion probable. In other words, it is claimed
that, on the assumption that the premises are true, the conclusion is unlikely to be false.

Revealing the logical form of a deductive argument helps in logical analysis and
evaluation. When we evaluate deductive arguments, we use the following concepts:
valid, invalid, sound and sound. A valid argument is one where, assuming that the
premises are true, it is impossible for the conclusion to be false. In other words, the
conclusion necessarily follows from the premises. An invalid argument is one where,
assuming that the premises are true, it is possible for the conclusion to be false. In other
words, a deductive argument in which the conclusion does not necessarily follow the
premises is an invalid argument. If the logical analysis shows that the deductive
argument is valid, and if the truth value analysis of the premises shows that they are all
true, then the argument is sound. If the deductive argument is invalid, or if at least one
of the premises is false (true value analysis), the argument is not sound.

2
A counterexample to a statement is evidence that shows that the statement is false,
and it concerns an analysis of truth value. The counterexample to the argument shows
the possibility that the premises assumed to be true do not necessarily make the
conclusion true. A single counterexample to a deductive argument is sufficient to show
that the argument is invalid.

To show that an argument form is invalid, we need only show that an argument in that
form can have all true premisses and a false conclusion. As with denying the antecedent,
we can do this abstractly by discussing the circumstances under which sentences of a
certain form are true or false. Alternatively, we can exhibit English
language argument that fits the form and obviously has true premisses and a false
conclusion. This latter method is called providing a counterexample.

When assessing inductive arguments, we use the following concepts: strong, weak,
cogent, and uncogent. A strong inductive argument is such that if the premises are
assumed to be true , the conclusion is probably true. In other words, if the premises are
assumed to be true, then the conclusion is unlikely to be false.

A. Statements and Arguments. Arguments are made up of sentences. The sentences


that state the evidence are premisses; the sentence being argued for is the conclusion.
An argument can have any number of premisses, but (by definition) it can have only one
conclusion.

“Inference” refers to the reasoning process expressed by an argument. In a good


argument, the conclusion is supported by the evidence given in the premise(s). What
distinguishes an argument from other collections of statements is its inferential nature.

B. Recognizing Arguments. Indicator words help us identify the elements of an


argument.

Conclusion indicators (such as “therefore,’” “so,” “it follows that”) alert you to the
appearance of a conclusion, while premise indicators (such as “since,” “because,” “it
follows from”) alert you to the appearance of a premise.
In each case, indicator words tell you that a conclusion or premise is about to be, or has
just been, asserted.

C. Arguments and Explanations. An explanation can sometimes be taken for an


argument, and vice versa. In addition, both arguments and explanations often use the
same indicator words.

The crucial distinguishing feature of an argument is that the truth of the conclusion is in
question. So, even when an explanation involves indicator words, there is no intent to
prove anything or settle some sort of issue: “Because you were late meeting me at the
restaurant for dinner, I went ahead and placed my order.” Here, an explanation is
offered for ordering food, an already accepted fact.

D. Truth and Logic. Truth value analysis refers to whether the individual statements in
the argument are accurate, correct, or true. Truth value analysis is applied to individual
statements.

Truth and Logic


Truth analysis: Determines whether the information in the premises is accurate,
correct, or true. Truth analysis is about statements.

3
Logical analysis: Determines the strength with which the premises support the
conclusion. Logical analysis is about arguments.

Logical analysis, refers to the strength with which the premises support the conclusion.
Logical analysis is applied to arguments.

E. Deductive and Inductive Arguments. Arguments fall into one of two types: A
deductive argument is one in which the conclusion follows necessarily from the
premises. In other words, under the assumption that the premises are true it is
impossible for the conclusion to be false.

An inductive argument is one in which the conclusion is probably true if the premises
are true. In other words, under the assumption that the premises are true it is
improbable for the conclusion to be false.

F. Deductive Arguments: Validity and Soundness. Deductive arguments are either valid
or invalid and sound or unsound.

A valid deductive argument is one in which it is impossible for the conclusion to be false
assuming that the premises are true. In other words, the conclusion follows necessarily
from the premises.

An invalid argument is one in which it is possible for the conclusion to be false, even if
the premises are true. In this case, the conclusion does not follow necessarily from the
premises.

A sound argument is valid, and its premises are actually true. An unsound argument is
either invalid or at least one of its premises are false. A convenient test of validity is the
counterexample method. A counterexample to a statement is evidence that shows the
statement is false. A counterexample to an argument shows the possibility that
premises assumed to be true do not make the conclusion necessarily true. It shows the
possibility that the conclusion does not follow necessarily from true premises. A single
counterexample to a deductive argument is enough to show that an argument is invalid.

Deductive Arguments: Validity and Soundness. Valid Argument: In a valid deductive


argument, it is impossible for the conclusion to be false assuming the premises are true.
If the premises actually are true, then the deductive argument is sound as well.

G. Inductive Arguments: Strength and Cogency. When we evaluate inductive


arguments, we use the following concepts: strong, weak, cogent, and uncogent. A
strong inductive argument is one such that if the premises are assumed to be true, then
the conclusion is probably true. In other words, it is improbable that the conclusion is
false if the premises are assumed to be true. A weak inductive argument is one such
that either (a) if the premises are assumed to be true, then the conclusion is probably
not true, or (b) a probably true conclusion does not follow from the premises. This latter
case typically occurs when the premises are simply irrelevant to the truth of the
conclusion.

An inductive argument is cogent when the argument is strong and the premises are
true. An inductive argument is uncogent if either or both of the following conditions
hold: the argument is weak, or the argument has at least one false premise.

H. Reconstructing Arguments. Whenever we begin to analyze an argument, whether or


not we agree with the conclusion, we should enlist the principle of charity. The principle
4
asserts that we should choose the reconstructed argument that gives the benefit of the
doubt to the person presenting the argument.

The principle of charity is also useful to reconstructing incomplete arguments. This type
of argument is called an enthymeme. Enthymemes are missing a conclusion or one or
more premises, or both.

There are several other types of incomplete or unclearly stated arguments and also
several ways in which unclear language leads to erroneous inferences. The principle of
charity is, once again, helpful in reconstructing or clarifying such arguments:
Rhetorical language: Occurs when we speak or write for dramatic or exaggerated
effect; when the language we employ may be implying things that are not explicitly said.
Rhetorical question: Occurs when a statement is disguised in the form of a question.
Rhetorical conditional: Occurs when a conditional statement is used to imply an
argument.

5
ASSESSMENT

Name:__________________________________________ Date:_______________
Year & Section:___________________________________ Rating:______________

Direction: Encircle the letter of the correct answer.

1. Identify the conclusion in the following argument.

My character would never betray his old friend and superior officer. This plot
twist is cheap and contrived. You need to rewrite this whole story, because if this
goes on, I’m going to quit the show.
a. My character would never betray his old friend and superior officer.
b. This plot twist is cheap and contrived.
c. You need to rewrite this whole story.
d. If this goes on, I’m going to quit the show.
2. Identify the conclusion in the following argument.

Wild hogs often damage newly planted seedlings and food crops by rooting, and
that also causes soil erosion. In addition, hogs compete with other animals that
also forage, usually crowding them out. Lastly, hogs are vectors for diseases
communicable to humans and livestock. Farmers should always take vigorous
action to reduce wild hog populations near their land.
a. Wild hogs often damage new plants and cause soil erosion.
b. Wild hogs compete with other foraging animals.
c. Wild hogs spread disease to humans and livestock.
d. Farmers should take vigorous action to reduce wild hog populations
near their land.
3. In the following argument, identify the main conclusion.

There will never be a fifth face added to Mount Rushmore. People are used to it
as it is. Moreover, political culture in the U.S. is more divisive than ever, so no
one will be able to agree on whom to add.
a. There will never be a fifth face added to Mount Rushmore.
b. People are used to Mount Rushmore as it is.
c. Political culture in the U.S. is more divisive than ever.
d. No one will be able to agree on whom to add.
4. Does the following passage contain an argument?

Knowledge is held to be successful cognition, the result of reflective processes


characterized by careful thinking and explicit defense of claims via reasoned
argument. For something to count as a reason, a thinker must (also) have reason
to believe it is true. This straightforwardly implies that knowledge must be based
in infinite series of justification.
a. Argument
b. b. No argument

5. Does the following passage contain an argument?

Although he began as a figure of unalloyed patriotism in the comic books of


World War II, since his revival in the mid-1960s, Captain America has more often
than not been a vehicle of critical reflection on the American Dream. Nowhere is
this tendency clearer than in his post-9/11 characterization.

6
a.Argument
b. No argument
6. Does the following passage contain an argument?

In retrospect, it seems like a mistake to have built two branch libraries in the
suburbs. While they may have seemed like good ideas in a booming economy 15
years ago, in the current recession it is a wasteful dispersion of limited public
resources.
a.Argument
b. No argument
7. Is the following passage an argument or an explanation?

There are several hundred different kinds of odorant receptors, each one with a
binding site shaped to accommodate a specific common odorant. We can,
however, discriminate between a wider range of odours than this, because each
odour is typically the result of a complex blend of different odorant molecules.
– Philip Ball, Stories of the Invisible
a.Argument
b. Explanation
8. Is the following passage an argument or an explanation?

Why would I invite a complete stranger into my home? Well, that’s not the issue.
Upon noticing that there was mud on his boots, I deduced that he must have
traveled some distance, since there is no running water anywhere nearby, and it
hasn’t rained anytime recently around here.
a.Argument
b. Explanation
9. Is the following passage an argument or an explanation?

I wake up every day and drag myself out into the world, enduring this dreadful
job and hateful people, so that I can make a future for my children that is gentler
and more peaceful than my own childhood was.
a. Argument
b. Explanation
10. Is the following argument best classified as deductive or inductive?

No book in English begins numbering its pages on a left-hand page. This is a book
in English, therefore it will begin its numbering on a right-hand page.
a.Deductive
b. Inductive
11. Is the following argument best classified as deductive or inductive?

Based on a survey of 2200 randomly selected likely voters, 56.2% indicate that
they will vote for the incumbent in the upcoming election. Therefore,
approximately 56% of the votes in the upcoming election will be for the
incumbent.
a.Deductive
b. Inductive
12. Choose which argument form correctly models the given argument, and which
substitutions create a counterexample that shows the argument is invalid.

7
All miracles are highly improbable events. Some highly improbable events are
lottery wins. So, some lottery wins are miracles.
a. All A are B. Some B are C. So, some A are C. Let A = aardvarks, B =
bumblebees, and C = cats.
b. All A are B. Some C are B. So, some C are A. Let A = asteroids, B =
things that drift, and C = Canadians.
c. All A are B. Some B are C. So, some C are A. Let A = asteroids, B =
things that drift, and C = floating things.
d. All A are B. Some B are C. So, some C are A. Let A = things that attack
humans, B = dangerous things, C = cats.
13. Choose which argument form correctly models the given argument, and which
substitutions create a counterexample that shows the argument is invalid.

No reality TV stars are normal people. Some Olympic athletes are not normal
people. So, some Olympic athletes are not reality TV stars.
a. No A are B. Some C are not B. So, some A are not C. Let A = animals, B
= rocks, and C = cats.
b. No A are B. Some B are not C. So, some C are not A. Let A = A-list
actors, B = poor people, and C = bankers.
c. No A are B. Some C are not B. So, some C are not A. Let A = animals, B
= rocks, and C = cats.
d. No A are B. Some C are not B. So, some C are not A. Let A = A-list
actors, B = poor people, and C = bankers.
14. Is the following inductive argument strong or weak?

Almost all dogs bark. I hear something barking. So it’s probably a dog.
a.Strong
b. Weak
15. Is the following inductive argument strong or weak?

Approximately 45% of women above the age of 20 live to be 85. Ann is 21, so
Ann will live to be at least 85.
a.Strong
b. Weak

8
LESSON II
LANGUAGE
TOPICS
1. The uses of language

LEARNING OUTCOMES
At the end of the lesson, student should be able to:
 Drill & sharpen students’ ability in establishing logical relations.
 Train and form students the appropriate technique for logical reasoning.

Language is a key argument. After all, the argument consists of words strung together to
form statements (sentences with a truth value), one of which is established by the other
or by others. This chapter focuses on how to use words. More specifically, the focus of
this chapter is on meaning.

TOPIC 1: THE USES OF LANGUAGE

A. Intension and Extension. We can study the meaning of definitions. More specifically,
we can look at the definition of the term: "a single word or a group of words that may
be the subject of a statement." Note that not all words are terms. Terms — common
names, proper names, and descriptive phrases — are not adjective-like words.

Intension and Extension. A term—a single words or group of words that can be the
subject of a statement—has meaning. We may specify that meaning by focusing on
characteristics, properties, or features associated with that term, or by focusing on how
that term is classified. When we say “cat,” for example, means “furry, four-legged,
meowing” creature, we focus on the term’s intensional meaning. When we say “cat”
means “feline animal,” we focus on the term’s denotative or extensional meaning.

Intension, we define it according to its characteristics. “Cat,” is a furry, four-legged,


meowing creature. Extension, we define it according to the class or collection of things
to which the term refers. “Cat” is a feline animal.

B. Using Intensional Definitions. When we define, we assign meaning to a word, phrase,


or symbol. So, we have that which is defined (the definiendum) and that which defines
(the definiens). Terms defined intensionally are often defined by synonyms, word
origins, operational feature, and genus and difference.

C. Using Extensional Definitions. Terms defined extensionally are often


defined ostensively, enumeratively, and by subclass.

D. Applying Definitions. Intensional and extensional definitions are not the only types of
definitions. Some other common types include stipulative, lexical, functional, precising,
theoretical, and persuasive.
9
E. Guidelines for Informative Definitions. The clearer and more exacting are our
definitions of words and use of those words in statements, the better our arguments
will be. There are eight guidelines which are meant to facilitate the construction of
informative definitions. These guidelines are helpful to the production or identification
of meaning, which in turn leads us closer to understanding:
1. An informative definition should use quotation marks appropriately.
2. An informative definition should include the essential meaning of a term.
3. An informative definition should not be too broad nor too narrow.
4. An informative definition should not be circular.
5. An informative definition should be affirmative and not negative.
6. An informative definition should not use ambiguous or vague language.
7. An informative definition should not use emotionally charged or figurative
language.
8. An informative definition should include a context whenever necessary.

F. Cognitive and Emotive Meaning. How we choose to use language says a lot about
what we mean or what we mean by our words. For example, we can choose to use
language cognitively or emotionally. The former is distinguished from the latter in terms
of language which attempts to describe or convey information rather than express
emotion. Both types of meaning can be found in value claims, which are judgments that
someone or something is moral or immoral, good or bad, or better or worse than
someone or something else.

Cognitive and Emotive Meaning. The difference between cognitive and emotive
meaning is the difference between fact and feeling. When we attempt solely to
describe, our focus is on cognitive meaning. When we express feelings—specifically
emotional states—our focus is on emotive meaning.

G. Factual and Verbal Disputes. It is important to distinguish between factual and oral
disputes. A factual dispute arises where there is disagreement over matters involving
facts. A verbal dispute arises when vague or ambiguous language results in a linguistic
misunderstanding. Since this kind of dispute is really a misunderstanding, we say it is
"merely" verbal.

10
ASSESSMENT

Name:__________________________________________ Date:_______________
Year & Section:___________________________________ Rating:______________

Direction: Encircle the letter of the correct answer.

1. Which of these properties is not connoted by the term “history”?


a. record
b. old
c. chronology
d. past

2. Which of these properties is not connoted by the term “machine”?


a. complicated
b. unemotional
c. instrument
d. constructed

3. Which of these properties is not connoted by the term “epic”?


a. heroic
b. story
c. legendary
d. cool

4. Which of the following are denoted by the term “feline”?


a. tiger
b. cheetah
c. puma
d. ocelot
e. all of the above

5. Which of the following are denoted by the term “sport”?


a. baseball, chess
b. soccer, poker
c. bridge, football
d. volleyball, tennis

6. Which of the following are denoted by the term “computer”?


a. MacBook Pro
b. MacBook Air
c. Acer Inspire
d. Dell Inspiron
e. all of the above

7. Name the thing denoted by the term “tallest structure in the world.”
a. Burj Kalifa
b. Taipei 101
c. Sears Tower
d. Tokyo Skytree

8. Identify the thing denoted by the term “first person to formulate equations
unifying electricity, magnetism, and light.”

11
a. Einstein
b. Newton
c. Maxwell
d. Marconi

9. Which one of the following names refers to a person denoted by the term “The
Three Stooges.”
a. Manny
b. Lou
c. Jack
d. Larry

10. Arrange the following series of terms in the order of decreasing extension: actor,
Cate Blanchett, mammal, organism, person
a. Cate Blanchett, actor, person, mammal, organism
b. organism, mammal, person, actor, Cate Blanchett
c. person, actor, Cate Blanchett, mammal, organism
d. organism, mammal, Cate Blanchett, actor, person

11. Arrange the following series of terms in the order of increasing extension: Anne
of Green Gables, bestseller, book, novel, writing.
a. Anne of Green Gables, bestseller, novel, book, writing
b. bestseller, book, writing, novel, Anne of Green Gables
c. novel, bestseller, book, writing, Anne of Green Gables
d. book, bestseller, writing, novel, Anne of Green Gables

12. Which type best describes the following definition?


Third world countries are countries like Botswana, Bangladesh, and Papua New
Guinea.
a. synonymous
b. word origin
c. operational
d. genus and difference
e. enumerative

13. Which type best describes the following definition?


A desire for a pet kitten was measured by the number of times the speaker
mentioned kittens in an approving or positive way, or the number of times the
speaker stared at kittens in a pet store window.
a. synonymous
b. word origin
c. operational
d. genus and difference
e. ostensive

14. Which type best describes the following definition?


The word “philosophy” originally referred to “the love of wisdom,” from the
Greek words for love and wisdom.
a. synonymous
b. word origin
c. operational
d. genus and difference
e. ostensive

12
15. Which type of definition from Section 2.D best describes the following?
As it purposely arouses sexual desire, it is pornography.
a. Stipulative
b. Lexical
c. Precising
d. Theoretical
e. Functional

16. Which type of definition from Section 2.D best describes the following?
“Minor” means anyone under the age of 18.
a. Stipulative
b. Lexical
c. Precising
d. Theoretical
e. Functional

17. Which type of definition from Section 2.D best describes the following?
Health care rationing will necessitate choices that decide when and whether life
support or treatment continues. These will be, in effect, death panels.
a. Stipulative
b. Lexical
c. Persuasive
d. Theoretical
e. Functional

18. Which of the following guidelines for informative definitions (Section 2.E) best
applies to this passage?
“Consciousness” means a tangled skein, the threads of our innermost yearnings
and their interconnections.
a. Too broad or too narrow
b. Emotional or figurative
c. Circular
d. Too vague or ambiguous

19. Which of the following guidelines for informative definitions (Section 2.E) best
applies to this passage?
“Communism” means the abolition of private property.
a. Negative when it could be affirmative
b. Too vague or ambiguous
c. Emotional or figurative
d. Not enough context

20. Which of the following guidelines for informative definitions (Section 2.E) best
applies to this passage?
“Humanity” means the animal that wears clothes.
a. Inappropriate use of quote marks
b. No essential meaning
c. Too vague or ambiguous
d. Circular

13
LESSON III
DIAGRAMS AND ANALYSIS

TOPICS
1. The Basics of Diagramming Arguments
2. Diagramming Extended Arguments

LEARNING OUTCOMES
At the end of the lesson, student should be able to:
 To identify the logical structure and relationship of the arguments.

Diagraming is a powerful method for understanding arguments. That's because the


premises and conclusions of the diagram show the relationship between all the parts of
the argument. Seeing these relationships helps us to understand the reasoning process.

TOPIC 1: THE BASICS OF DIAGRAMMING ARGUMENTS

The diagram arguments reveal the relationship between the premises and the premises
and the conclusion. Another way to say this is that the diagram is a kind of map that
allows you to see how you move from the premises to the conclusion.

 A diagram of an argument is like a map showing the relationship between the


premise(s) and conclusion.
 An argument consisting of only one premise (1) and conclusion (2) is
diagrammed:

The premises can support the conclusion independently or independently. Independent


premises support the conclusion that, if one premise were to be false, the truth of the
conclusion would not be affected. Dependent premises support the conclusion jointly,
that is to say, they support the conclusion in such a way that they act together.

Arguments are diagramed using numbers, arrows, braces, and brackets as needed. Each
argument statement is numbered. The arrows indicate the direction of the premise-
conclusion relationship, and the braces reflect the dependent premises.

A convergent diagram shows an argument with independent premises.

14
A linked diagram reflects an argument with dependent premises.

A divergent diagram reflects a single premise supporting independent conclusions.

A serial diagram reflects a conclusion from one argument serving as a premise in a


second argument.

TOPIC 2: DIAGRAMMING EXTENDED ARGUMENTS

Extended arguments often require diagrams that combine two or more of our
diagramming techniques. For example, the following extended argument diagram
contains linked, convergent, divergent, and serial diagrams.

Diagramming arguments helps us to see how the parts of an argument relate to each
other. Premises can independently or jointly support a conclusion. In addition, there
may be chains of reasoning, where the conclusion of one piece of reasoning serves as a
premise in a larger argument.

15
ASSESSMENT

Name:__________________________________________ Date:_______________
Year & Section:___________________________________ Rating:______________

Set 1. Direction: Encircle the letter of the correct answer.

1. Diagram the following argument using the techniques from Section 3A.
1
According to Freud, men only desire fame, wealth, and women.
However, 2 Kant did not seek fame, lived frugally, and he never married.
So 3 Freud’s theory does not explain all men.

a.

b.

c.

d.

2. Diagram the following argument using the techniques from Section 3A.
1
Mythical thinking has just as much a claim to being practically effective as
scientific thinking does. 2 When human beings thought in mythic terms, they
invented fire and farming. 3 The mythic mind domesticated animals and bred
new species of plants and animal. 4 They invented, from nothing, the wheel and
other fundamental tools from which all later science grew.

a.

b.

c.

d.

16
Incomplete Arguments
3. For the following enthymeme, use the principle of charity to supply either the
missing premises or the missing conclusion to make the argument valid or
strong.

Eating meat is natural, so it’s unobjectionable to eat meat.


a. No one should complain about eating meat. (premise)
b. Human beings are designed to eat meat. (premise)
c. If people did not eat meat, they would suffer malnutrition. (premise)
d. Natural activities are unobjectionable. (premise)

4. For the following enthymeme, use the principle of charity to supply either the
missing premises or the missing conclusion.

She looks like Superwoman. Any time someone has looked like Superwoman in
the history of our sport they have later been found guilty of doping.
– U.S. Olympic swimming coach John Leonard

a. If someone looks like Superwoman in our sport, then they are doping.
(premise)
b. If someone is doping, then they look like Superwoman. (premise)
c. She is guilty of doping. (conclusion)
d. The history of our sport consistently shows super performances are
the result of doping. (premise)

Strengthening or Weakening Arguments


5. Consider the argument “the gas pump automatically stopped pumping gas, so
the car’s gas tank is full.” Would the following claim strengthen or weaken that
argument?

The car has an uncommonly large gas tank.

a. Strengthen
b. Weaken

Reconstructing Arguments Based on Rhetorical Language


6. Choose the reconstructed conclusion of the following rhetorical language which
seems the best interpretation.

If all American philosophers were gathered into one place and an atomic bomb
exploded on them, American society would remain totally unaffected. No one
would notice any difference, and there would be no gap, no vacuum in the
intellectual economy that would require plugging.
– Ernest Gellner, The Devil in Modern Philosophy

a. American philosophy is unimportant to society and intellectual life.


b. American philosophers should be destroyed with atomic weapons.
c. No one would notice if American philosophy ceased to exist.
d. American philosophy should be defended against its opponents.

17
LESSON IV
FALLACIES

TOPICS
1. Fallacies Based on Personal Attacks
2. Fallacies Based Emotional Appeal
3. Weak Inductive Argument Fallacies
4. Fallacies of Unwarranted Assumption

LEARNING OUTCOMES
At the end of the lesson, student should be able to:
 Evaluate faulty reasoning and support charge of.

Arguments fail when the conclusion does not follow necessarily or probably from the
premises. A formal fallacy is one that may be identified by merely examining the form
or structure of an argument. Informal fallacies are those that can be detected only by
examining the content of an argument.

Fallacies are instances of incorrect reasoning, the premises of which do not provide
good grounds for believing the conclusion. You will be less likely to commit these
mistakes by studying flaws. A fallacy is a defect in an argument that arises from either a
mistake in reasoning or the creation of an illusion that makes a bad argument appear
good.

TOPIC 1: FALLACIES BASED ON PERSONAL ATTACKS

The following four fallacies are based on personal attacks against the person making the
argument, not on the merit of the argument itself.

1. Ad hominem abusive: The fallacy is distinguished by an attack on alleged character


flaws of a person instead of the person’s argument.
2. Ad hominem circumstantial: When someone’s argument is rejected based on the
circumstances of the person’s life.
3. Poisoning the well: The fallacy occurs when a person is attacked before she has a
chance to present her case.
4. Tu quoque: The fallacy occurs when a person attempts to avoid the issue at hand by
claiming the other person is a hypocrite whose actions and words are inconsistent.

Fallacies based on personal attacks occur when someone attacks you instead of
examining your argument. The attack is not relevant to the position you advanced. Most
often, such attacks are successful because they prey on our emotional vulnerabilities. In
other words, we get distracted by the vehemence of the attack itself, rather than realize
that the attacker has not addressed the content or structure of our position.

18
TOPIC 2: FALLACIES BASED EMOTIONAL APPEAL

The following three fallacies are based on emotional appeal, relying on the arousal of a
strong emotional state or psychological reaction to get us to accept a conclusion.

1. Appeal to the people: The fallacy occurs when an argument manipulates a


psychological need or desire for belonging to a group so a reader or listener will accept
the conclusion.
2. Appeal to pity: The fallacy results from an exclusive reliance on a sense of pity or
mercy for support of a conclusion. occurs when an arguer attempts to support a
conclusion by merely evoking pity from the reader or listener.
3. Appeal to fear or force: A threat of harmful consequences (physical or otherwise)
used to force acceptance of a course of action that would otherwise be unacceptable.
occurs whenever an arguer presents a conclusion to another person and tells that
person either implicitly or explicitly that some harm will come to him or her if he or she
does not accept the conclusion.

TOPIC 3: WEAK INDUCTIVE ARGUMENT FALLACIES

Generalization fallacies occur when there is a mistaken application of a generalization


such that the premise(s) supply only very weak support for the truth of the conclusion.

1. Rigid application of a generalization: When a generalization or rule is inappropriately


applied to the case at hand. The fallacy results from the belief that a generalization or a
rule is universal (meaning it has no exceptions).
2. Hasty generalization: An argument that relies on a small sample that is unlikely to
represent the population.
3. Composition: There are two forms of the fallacy: (1) The mistaken transfer of an
attribute of the individual parts of an object to the object as a whole. (2) The mistaken
transfer of an attribute of the individual members of a class to the class itself.
4. Division: There are two forms: (1) The mistaken transfer of an attribute of an object
as a whole to the individual parts of the object. (2) The mistaken transfer of an attribute
of a class to the individual members of the class.
5. Biased sample: An argument that uses a nonrepresentative sample as support for a
statistical claim about an entire population.

Generalization fallacies rely on a mistaken use of the principles behind making a


generalization. Someone may assign an unpleasant characteristic based on a stereotype
of a certain group, for example.

19
False Cause Fallacies occur when a causal connection is assumed to exist between two
events when none actually exists, or when the assumed causal connection is unlikely to
exist.

6. Post hoc: The fallacy occurs from the mistaken assumption that just because one
event occurred before another event, the first event must have caused the second
event. Subtypes of this fallacy are Coincidence, resulting from the accidental connection
between two events, and the Common Cause fallacy, which occurs when one event is
believed to cause a second event, when in fact both events are the result of a common
cause.
7. Slippery slope: An argument that attempts to connect a series of occurrences such
that the first link in a chain leads directly to a second link and so on, until a final
unwanted situation is said to be the inevitable result.

False cause fallacies attempt to connect two events as cause–effect when they
may have occurred together by coincidence or have been caused by a third, common
cause.

A slippery slope fallacy attempts to connect a series of events causally in such a


way as to lead inevitably to an undesirable result.

TOPIC 4: FALLACIES OF UNWARRANTED ASSUMPTION OR DIVERSION

Fallacies of unwarranted assumption assume the truth of some unproved or


questionable claim; when the assumptions and lack of support are exposed, the weak
points of the argument are exposed.

1. Begging the question: In one type, the fallacy occurs when a premise is simply
reworded in the conclusion. In a second type, called circular reasoning, a set of
statements seem to support each other with no clear beginning or end point. In a third
type, the argument assumes certain key information that may be controversial or is not
supported by facts.
2. Complex question: The fallacy occurs when a single question actually contains
multiple parts and an unestablished hidden assumption.
17. Appeal to ignorance: An argument built on a position of ignorance claims either that
(1) a statement must be true because it has not been proven to be false or (2) a
statement must be false because it has not been proven to be true.
3. Appeal to an unqualified authority: An argument that relies on the opinions of
people who either have no expertise, training, or knowledge relevant to the issue at
hand, or whose testimony is not trustworthy.
4. False dichotomy: The fallacy occurs when it is assumed that only two choices are
possible, when in fact others exist.

20
Fallacies of diversion occur when the meanings of terms or phrases are changed
(intentionally or unintentionally) within the argument, or when our attention is
purposely (or accidentally) diverted from the issue at hand.

5. Equivocation: The fallacy occurs when the conclusion of an argument relies on an


intentional or unintentional shift in the meaning of a term or phrase in the premises.
6. Straw man: The fallacy occurs when an argument is misrepresented in order to create
a new argument that can be easily refuted. The new argument is so weak that it is
“made of straw.” The arguer then falsely claims that his opponent’s real argument has
been defeated.
7. Red herring: The fallacy occurs when someone completely ignores an opponent’s
position and changes the subject, diverting the discussion in a new direction.
8. Misleading precision: A claim that appears to be statistically significant but is not.
9. Missing the point: When premises that seem to lead logically to one conclusion are
used instead to support an unexpected conclusion.

Recognizing Fallacies in Ordinary Language. Being aware of the ways in which reasoning
goes wrong aids us in recognizing errors that occur in beautifully written or otherwise
inspired passages. It is especially helpful to us when we are confronted with a topic to
which we have an emotional connection or pre-existing belief. It’s difficult to be open-
minded, that is, to be ready to have your beliefs challenged. More difficult still is when
you are the one who has to challenge yourself!

21
ASSESSMENT

Name:__________________________________________ Date:_______________
Year & Section:___________________________________ Rating:______________

Direction: Encircle the letter of the correct answer.

1. Which fallacy of relevance best describes the following passage?

When my back aches after a long workout, I turn to Xoan’s for back pain relief.
Xoan’s Back Pain Relief works. I ought to know, I’m a seven-time all-pro football
player.

a. Ad hominem
b. Tu quoque
c. Appeal to unqualified authority
d. Appeal to pity
e. Appeal to force

2. Which fallacy of relevance best describes the following passage?

You can’t believe what Romney says about protecting religious rights—he’s a
Mormon.

a. Ad hominem
b. Tu quoque
c. Appeal to the people
d. Appeal to pity
e. Appeal to force

3. Which fallacy of relevance best describes the following passage?

You can be sure Romney’s tax plan is not going to benefit the country: he’s a
multi-millionaire and stands to benefit from extending the Bush tax cuts.

a. Ad hominem
b. Tu quoque
c. Appeal to the people
d. Appeal to pity
e. Appeal to force

4. Which fallacy of relevance best describes the following passage?

Professor, I know I failed the exam. But I really need to pass this class and if I
don’t pass, then I’ll have to retake the course and won’t be able to graduate on
time. Please allow me to retake the final exam.

a. Ad hominem
b. Tu quoque
c. Appeal to the people
d. Appeal to pity
e. Appeal to force

5. Which fallacy of relevance best describes the following passage?


22
You want proof that democracy is the best form of government? Well, that’s
why ancient Athens was a democracy. And so is America.

a. Ad hominem
b. Tu quoque
c. Appeal to the people
d. Appeal to pity
e. Missing the point

Fallacies of Unwarranted Assumption

6. Which fallacy of unwarranted assumption best describes the following passage?

What is the meaning of life? This simple question continues to vex humanity,
from the most erudite philosopher to the common man. No answer has yet
satisfied all, or even a majority, of those who have reflected upon it.

a. Accident
b. Begging the question
c. Complex question
d. Biased sample
e. Post hoc fallacy

7. Which fallacy of unwarranted assumption best describes the following passage?

I know that people say that the president has a 10% lead among likely voters in
our state. But I have over two hundred local friends on Facebook and almost
none of them are going to vote for the president. So I think the polls are wrong; I
bet the president will lose our state in a landslide.

a. Accident
b. Begging the question
c. Common cause fallacy
d. Biased sample
e. Post hoc fallacy

8. Which fallacy of unwarranted assumption best describes the following passage?

That Bach’s musical greatness is without peer cannot be doubted, as his talents
and genius far outshine that of all his rivals, even Beethoven and Mozart.

a. Accident
b. Begging the question
c. Common cause fallacy
d. Biased sample
e. Post hoc fallacy

Fallacies of Ambiguity or Diversion

9. Which fallacy of ambiguity or diversion best fits the following passage?

23
Evolution is a theory. But if it’s just a theory, then it is little more than an
educated guess. Students should not be taught that a guess is true. So schools
should not teach evolution, or at best they should teach it as one possible theory
among others, like creationism.

a. Equivocation
b. Amphiboly
c. Composition
d. Division
e. Emphasis

10. Which fallacy of ambiguity or diversion best fits the following passage?

Good sex doesn’t happen on its own. The pleasure, satisfaction, and intimacy of
sex involves more than just two people sleeping together in the same bed.

a. Equivocation
b. Amphiboly
c. Composition
d. Division
e. Emphasis

11. Which fallacy of ambiguity or diversion best fits the following passage?

Hey, look at this headline. It says, “Prostitutes Appeal to Pope.” Wow, I wouldn’t
have thought he would suddenly become so open-minded.

a. Equivocation
b. Amphiboly
c. Composition
d. Division
e. Emphasis

Recognizing Fallacies

12. Which fallacy best fits the following passage?

This salad sounds delicious. It’s made from fresh arugula, which I love, and has
dried cranberries and candied pecans, both of which I also love, some mild goat
cheese (yum!), and a vinaigrette dressing, which is my favorite dressing. So it’s
certain I’ll love this salad.

a. Ad hominem
b. Begging the question
c. Composition
d. Division
e. False dilemma

13. Which fallacy best fits the following passage?

Who is the most glamorous movie star in Hollywood today: Angelina Jolie or
Anne Hathaway?

a. Ad hominem
b. Begging the question

24
c. Coincidence
d. Division
e. False dichotomy

14. Which fallacy best fits the following passage?

Just before the class ends, the clock bell tolls three times. Every time this
happens, the class gets let out around five minutes later. And the class never
ends without the bell tolling. So the clock bell tolling three times causes the class
to end.

a. Ad hominem
b. Begging the question
c. Common cause
d. Division
e. False dichotomy

15. Which fallacy best fits the following passage?

Many Christians have fallen for the lie of believing in evolution. But if God didn’t
create the world just as the Bible says, then we can’t believe anything in
Scripture. So God must have created the world just as the Bible says.

a. Amphiboly
b. Begging the question
c. Coincidence
d. Division
e. False dichotomy

25
LESSON V
CATEGORICAL PROPOSITIONS

TOPICS
1. Types of categorical propositions.
2. Quantity, Quality, and Distribution
3. Existential Import
4. The Modern Square of Opposition
5. Conversion, Obversion, and Contraposition in the Modern Square
6. The Traditional Square of Opposition
7. Conversion, Obversion, and Contrapositive in the Traditional Square
8. Translating Ordinary Language into Categorical Propositions

LEARNING OUTCOMES
At the end of the lesson, student should be able to:
 Prepare students to critically analyse any philosophical claim.
 Train students to verify the truthfulness of any claim.

A categorical proposition is a proposition that relates to two classes of objects.


Categorical proposals contain both a subject and a predicate term. The subject-matter
comes first in the form of a standard categorical proposition. The term predicate comes
second in the standard form of a categorical proposition.

A standard-form categorical proposition has quantity and quality, and a specific method
of distribution for a given subject or term (or both). "Universal" and "particular" shall
refer to the quantity of a categorical proposition. 'Affirmative' and 'negative' refer to the
quality of a categorical proposition. The words "all," "no," and "some" are referred to as
"quantifiers." They 're telling us the extent of class inclusion or exclusion. The words
"are" and "are not" are referred to as "copula." They are simply forms of "to be" and
serve to link (to "couple") the subject class to the predicate class. If a categorical
proposition asserts something definite about every member of a class, then the term
designating that class is said to be distributed. On the other hand, if the proposition
does not assert something definite about every member of a class, then the term
designating that class is said to be undistributed.

TOPIC 1: TYPES OF CATEGORICAL PROPOSITIONS

Categorical propositions are statements about classes of things. A class is a group of


objects. There are two class terms in each categorical proposition, a subject class (S) and
a predicate class (P).

There are four types of categorical proposition:


 A-proposition: Asserts that the entire subject class is included in the predicate
class.
o Standard-form of the A-proposition: All S are P.

26
o This is the universal affirmative proposition.
 I-proposition: Asserts that at least one member of the subject class is included in
the predicate class.
o Standard form of the I-proposition: Some S are P.
o This is the particular affirmative proposition.
 E-proposition: Asserts that the entire subject class is excluded from the predicate
class.
o Standard-form of the E-proposition: No S are P.
o This is the universal negative proposition.
 O-proposition: Asserts that at least one member of the subject class is excluded
from the predicate class.
o Standard-form of the O-proposition: Some S are not P.
o This is the particular negative proposition.

TOPIC 2: QUANTITY, QUALITY, AND DISTRIBUTION

Categorical propositions can be viewed in terms of quantity (universal or particular),


quality (affirmative or negative) and whether or not a class is distributed.

If a categorical proposition asserts something about every member of a class, then the
term designating that class is said to be distributed. If a proposition does not assert
something about every member of a class, then the term designating that class is said to
be undistributed. In summary:

All S are P. The subject term is distributed; the predicate class is not.
Some S are P. Neither the subject nor predicate term is distributed.
No S are P. Both the subject and predicate terms are distributed.
Some S are not P. The subject term is not distributed; the predicate class is distributed.

TOPIC 3: EXISTENTIAL IMPORT

A proposition has existential import if it presupposes the existence of certain kinds of


objects.

Under the traditional interpretation of categorical propositions, the universal A- and E


propositions may be assumed to have existential import. This leads to problems
determining truth value when the classes referred to in the propositions lack any
members:
“All unicorns are mammals.”
“No unicorns are reptiles.”

Under the modern interpretation of categorical propositions, the universal A- and E


propositions are translated as conditional statements, thus resolving questions
concerning the existence of the members of a class:
“All unicorns are mammals.” becomes “If something is a unicorn, then that thing is a
mammal.”
“No unicorns are reptiles.” becomes “If something is a unicorn, then that thing is a not a
reptile.”

27
The particular I- and O-propositions in both the traditional and modern interpretations
of categorical propositions have existential import.

TOPIC 4: THE MODERN SQUARE OF OPPOSITION AND VENN DIAGRAMS

The square of opposition shows us the logical inferences (immediate inferences) we


can make from one proposition type (A, I, E, and O) to another.
Two propositions are said to be contradictories when both cannot be true at the same
time and both cannot be false at the same time. A and O propositions are contradictory,
and E and I propositions are contradictories.

Venn Diagrams
Since the modern interpretation does not assume existential import for A- and E
propositions, their Venn diagrams picture this status with circles and shaded (empty)
areas:

Venn diagrams for the two particular claims include an x representing existence:

TOPIC 5: CONVERSION, OBVERSION, AND CONTRAPOSITION IN MODERN SQ.

Conversion, obversion, and contraposition are further immediate inferences we can


make between one categorical proposition and another.

An immediate argument can be created by interchanging the subject and predicate


terms of a given categorical proposition, a process called conversion.

Under the modern interpretation, conversion is valid only for E- and I-propositions:
 “No S are P” is logically equivalent to “No P are S.”
 “Some S are P” is logically equivalent to “Some P are S.”

An immediate argument can also be formed by obversion, in which (1) the quality of the
given proposition is changed, and (2) the predicate term is replaced with its class
complement (the set of objects that do not belong to a given class).

Under the modern interpretation, obversion is valid for all four proposition types:
 “All S are P” is logically equivalent to “No S are non-P.”
 “No S are P” is logically equivalent to “All S are non-P.”
 “Some S are P” is logically equivalent to “Some S are not non-P.”
 “Some S are not P” is logically equivalent to “Some S are non-P.”

28
One more type of immediate argument, contraposition, is formed by replacing the
subject term of a given proposition with the complement of its predicate term, and then
replacing the predicate term of the given proposition with the complement of its subject
term.

Under the modern interpretation, contraposition is valid only for A- and O-propositions:
“All S are P” is logically equivalent to “All non-P are non-S.”
“Some S are not P” is logically equivalent to “Some non-P are not non-S.”

TOPIC 6: THE TRADITIONAL SQUARE OF OPPOSITION AND VENN DIAGRAMS

Because the Traditional Square of Opposition assumes existential import, the Venn
diagram for a universal claim will reflect the possible existence of at least one member
in the subject class for an A-proposition, and at least one member if the subject or
predicate class for an E-proposition (since, by conversion, “No S is P” is logically
equivalent to
“No P is S”).

Venn diagrams for the particular propositions are the same as for the modern
interpretation.

TOPIC 7: CONVERSION, OBVERSION, AND CONTRAPOSITIVE IN TRADITIONAL

In the traditional square of opposition, as in the modern square, Conversion is valid for
E- and I-propositions and never valid for O-propositions. However, under the
assumption of existence, Conversion by limitation is valid for A-propositions in the
traditional interpretation.

Obversion is valid for all four categorical proposition types, as in the modern
interpretation.

Contraposition is valid for A- and O-propositions and never valid for I-propositions, as in
the modern interpretation. However, under the assumption of existence, Contraposition
by limitation is valid for E-propositions in the traditional interpretation.

TOPIC 8: TRANSLATING ORDINARY LANGUAGE INTO CATEGORICAL PROPOSITIONS

Ordinary language rarely presents categorical propositions in standard-form. We must


take pains, then, to interpret a given ordinary language proposition so that we
accurately translate it into standard-form. The following are ways to organize our
thinking about how to construct a standard-form categorical proposition (quantity,
quality, subject class, copula, predicate class):

1. Missing Plural Nouns

29
a. Since every categorical proposition involves two classes of objects, it is
important to identify two nouns.
2. Nonstandard Verbs
a. Since every categorical proposition involves a copula, it is important
to identify and replace the connecting verb with “are.”
3. Singular Propositions
a. Propositions about individuals are always translated
as universal propositions.
4. Adverbs and Pronouns
a. Since every categorical proposition involves a quantifier, it is important
to identify and replace adverbs that describe places or times as
reflecting quantity.
b. Since every categorical proposition involves two classes of objects, it is
important to identify and replace unspecified nouns.
5. “It Is False That…”
a. Since there are two ways a categorical proposition can be negative (“No S
are P,” and “Some S are not P”), it is important to identify and rewrite
phrases expressing negation.
6. Implied Quantifiers
a. Since every categorical proposition involves a quantifier (universal or
particular), it is important to make quantifiers explicit.
7. Nonstandard Quantifiers
a. Since every categorical proposition involves a quantifier (universal or
particular), it is important to rewrite the quantifier in standard-form.
8. Conditional Statements
a. “If…then” statements should be rewritten as universal categorical
propositions, with the phrase after “if” appearing on the left side of the
copula, and the phrase after “then” (expressly stated or implied)
appearing on the right side of the copula.
9. Exclusive Propositions
a. All exclusive propositions should be rewritten so that the exclusive
category appears in the predicate position.
10. “The Only”
a. All expressions of “the only” should be rewritten so that the class
appearing after “the only” is expressed as the subject class of the
categorical proposition.
11. Propositions Requiring Two Translations
a. Since every categorical proposition contains two and only two classes of
things, whenever an ordinary language proposition contains more than
two classes of object, there will be more than one categorical
proposition.

EXPLORE

Categorical Propositions

Quantity, Quality, and Distribution

30
Distribution refers to whether a proposition asserts something about every member of a
class (distributed) or not every member of the class (undistributed).

Remember them like this: Universals (A, E) distribute subjects, and negations (E, O)
distribute predicates:

SUPN Subjects—Universals Predicates—Negatives

Existential Import
A proposition has existential import if it presupposes the existence of certain kinds of
objects. Under the traditional interpretation, a universal proposition contained the
assumption of the existence of at least one thing in the subject class. Under the modern
interpretation, universal propositions are equated with conditional statements. Thus,
“All S are P” becomes logically equivalent to “If it’s an S, then it’s a P” in the modern
interpretation.

The Modern Square of Opposition and 5F The Traditional Square of Opposition


In the modern square of opposition, we only have necessary inferences between
contradictories:

This means that A and O cannot both be false or both true at the same time, and E and I
cannot both be false or both true at the same time.

 In the traditional square of opposition, the following applies:

1. A and O, and E and I are contradictories.

31
In addition:
2. Universal claims (A and E) are contraries. A and E cannot both be true at the same
time, but both could be false. Hence, if you assume one universal claim is false, you
cannot make a necessary inference about its contrary.
3. Particular propositions (I and O) are subcontraries. I and O cannot both be false at the
same time, but both could be true. Hence, if you assume one particular proposition is
true, you cannot make a necessary inference about its subcontrary.
4. Truth flows downward and falsity flows upward from a super- to a subaltern, and from
a sub- to a superaltern. So, for example, a true A proposition yields a true I-proposition,
while a false I-proposition yields a false A-proposition. Notice there is no valid inference
you can make about
truth and falsity from a false A-proposition to an I-proposition or from a true I-
proposition to an A-proposition. The same goes for the E and O propositions.

32
ASSESSMENT

Name:__________________________________________ Date:_______________
Year & Section:___________________________________ Rating:______________

Direction: Encircle the letter of the correct answer.

Analyze the following categorical propositions by doing the following: (1) Identify the
subject and predicate of each proposition; (2) identify the categorical proposition as
either A (All S are P), E (No S are P), I (Some S are P), or O (Some S are not P).

1. All humans are mammals.

a. Subject term: humans; Predicate term: mammals. This is an example


of an I-proposition.
b. Subject term: mammals; Predicate term: humans. This is an example
of an A-proposition.
c. Subject term: mammals; Predicate term: humans. This is an example
of an I-proposition.
d. Subject term: humans; Predicate term: mammals. This is an example
of an A-proposition.

2. No friends are people you should lie to.

a. Subject term: friends; Predicate term: people you should lie to. This is
an example of an O-proposition.
b. Subject term: friends; Predicate term: people you should lie to. This is
an example of an E-proposition.
c. Subject term: people you should lie to; Predicate term: friends. This is
an example of an O-proposition.
d. Subject term: people you should lie to; Predicate term: friends. This is
an example of an E-proposition.

3. Some dogs are not vicious animals.

a. Subject term: dogs; Predicate term: vicious animals. This is an example


of an I-proposition.
b. Subject term: vicious animals; Predicate term: dogs. This is an example
of an O-proposition.
c. Subject term: dogs; Predicate term: vicious animals. This is an example
of an O-proposition.
d. Subject term: dogs; Predicate term: vicious animals. This is an example
of an E-proposition.

The following categorical propositions are to be analyzed in the following three ways:
(1) the correct quantity ( universal or particular); (2) the
correct quality ( affirmative or negative); (3) the correct distribution (subject term
distributed; predicate term distributed; both terms distributed; or neither term
distributed).

Choose the correct answer.

33
4. No reputable companies are sexist companies.

a. Universal negative; Subject term distributed; Predicate term


distributed.
b. Particular negative; Subject term distributed; Predicate term
undistributed.
c. Universal negative; Subject term undistributed; Predicate term
distributed.
d. Particular affirmative; Subject term distributed; Predicate term
distributed.

5. No vegans are carnivores.

a. Universal affirmative; Subject term distributed; Predicate term


distributed.
b. Universal negative; Subject term undistributed; Predicate term
distributed.
c. Universal negative; Subject term distributed; Predicate term
distributed.
d. Universal negative; Subject term distributed; Predicate term
undistributed.

6. Some “Cats” are musicals on Broadway.

a. Universal affirmative; Subject term distributed; Predicate term


undistributed.
b. Particular affirmative; Subject term undistributed; Predicate term
undistributed.
c. Universal affirmative; Subject term undistributed; Predicate term
undistributed.
d. Particular affirmative; Subject term distributed; Predicate term
distributed.

Choose the Venn diagram that represents the correct relationship of the given
proposition.
7. Some reality shows are scripted events.

Let S = reality shows, and P = scripted events

a.

b.

34
c.

d.

8. Some animals are not tame animals.

Let S = animals, and P = tame animals

a.

b.

c.

d.

9. All hipsters are people who ride fixies.

Let S = hipsters, and P = people who ride fixies

a.

b.

35
c.

d.

10. All friends are people who respect you.

Let S = friends, and P = people who respect you

a.

b.

c.

d.

11. No dogs are cats.

Let S = dogs, and P = cats

a.

b.

36
c.

d.

Draw Venn diagrams for the following immediate arguments. Determine whether each
argument is valid or invalid using both the modern and traditional interpretations.

Choose the correct answer.

12. No actors are honest people. Therefore, some actors are honest people.

a. Valid under the traditional interpretation only.


b. Valid under the modern interpretation only.
c. Valid under both interpretations.
d. Invalid under both interpretations.

13. Some plants are conifers. Therefore, no plants are conifers.

a. Valid under the traditional interpretation only.


b. Valid under the modern interpretation only.
c. Valid under both interpretations.
d. Invalid under both interpretations.

14. All dogs are friendly animals. Therefore, some dogs are friendly animals.

a. Valid under the traditional interpretation only.


b. Valid under the modern interpretation only.
c. Valid under both interpretations.
d. Invalid under both interpretations.

Determine the correct standard form proposition translation of the following ordinary
language statements.

15. Some thinkers ruminate.

a. Some thinkers are not ruminators.


b. Some thinkers ruminate.
c. Some thinkers do not ruminate.
d. Some thinkers are ruminators.

37
LESSON VI
SYLLOGISM

TOPICS
1. Standard-Form Categorical Syllogism
2. Diagramming in the Modern Interpretation
3. Diagramming in the Traditional Interpretation
4. Mood and Figure
5. Rules and Fallacies
6. Ordinary Language Arguments
7. Enthymemes
8. Sorites

LEARNING OUTCOMES
At the end of the lesson, student should be able to:
 Drill and prepare students to the syllogistic process of reasoning
 Demonstrate how valid syllogism works.

A syllogism is a deductive argument with precisely two premises and a conclusion. A


categorical syllogism is made entirely of categorical proposals. It contains three different
terms, each of which is used twice. The main term is the predicate of the conclusion of a
categorical syllogism. The major term is the predicate of the conclusion of a categorical
syllogism. The minor term is the subject of the conclusion of a categorical syllogism. The
middle term is the term that occurs only in the premises of a categorical syllogism.

Following the structure and naming convention of categorical terms, the major premise
is the first premise of a categorical syllogism. The major premise contains the major
term. The minor premise the second premise of a categorical syllogism contains the
minor term.

In order to be a standard-form categorical syllogism, three requirements must be met:


1) All three statements must be standard-form categorical propositions.
2) The two occurrences of each term must be identical and have the same sense.
3) The major premise must occur first, the minor premise second, and the
conclusion last.

The mood of a categorical syllogism consists of the type of categorical propositions


involved (A, E, I, or O) and the order in which they occur. The middle term can be
arranged in the two premises in four different ways. These placements determine the
figure of the categorical syllogism.
There are six rules for standard-form categorical syllogisms:
1) The middle term must be distributed in at least one premise.
2) If a term is distributed in the conclusion, then it must be distributed in a
premise.
3) A categorical syllogism cannot have two negative premises.
4) A negative premise must have a negative conclusion.

38
5) A negative conclusion must have a negative premise.
6) Two universal premises cannot have a particular conclusion.

An enthymeme is an incomplete argument—either at least one premise is missing, or


the conclusion is missing. Sorites is a special type of enthymeme in which the missing
parts are intermediate conclusions each of which, in turn, becomes a premise in the
next link in the chain.

TOPIC 1: STANDARD-FORM CATEGORICAL SYLLOGISMS

A categorical syllogism is an argument containing three categorical propositions: two


premises and one conclusion. The most methodical way to study categorical syllogisms
is to learn how to put them in standard-form, which looks like:

Major premise
Minor premise
Conclusion

As you can see, in order to put an argument in standard-form, you need to know
the major premise and minor premise. The major premise contains the major term. The
major term is the predicate of the conclusion. The minor premise contains the minor
term. The minor term is the subject of the conclusion:

Major premise (contains the major term)


Minor premise (contains the minor term)
Minor term, copula, major term

Notice that the major and minor terms have nothing to do with quantity or quality.
Moreover, in the major premise, the major term may appear in the subject position, or
it may appear in the predicate position. The same is true for the minor term.

The premises also contain the middle term. This term does not appear in the
conclusion, but it appears once in each premise:
Major premise (contains the major term and the middle term)
Minor premise (contains the minor term and the middle term)
Minor term, copula, major term

TOPIC 2: DIAGRAMMING IN THE MODERN INTERPRETATION

Whereas individual categorical propositions contain two classes of things, a categorical


syllogism contains three classes. That means that we use three circles to create a Venn
diagram for a categorical syllogism:

39
When you diagram a categorical syllogism, the goal is to see whether the premises
support the conclusion to yield a valid argument. To test a categorical syllogism by way
of a Venn diagram involves diagramming only the premises. Once you diagram the
premises, you look to see if the conclusion appears. If not, the argument is invalid.

Steps for diagramming the premises of a categorical syllogism in the modern


interpretation:

1. If one of the premises is a universal proposition, diagram it first. (If both


premises are universal, it does not matter which one you diagram first.) This is
because you want to eliminate any place where an x, which represents a
particular proposition, cannot go.
2. Diagram the premise without regard to the third circle, since this is not relevant
to the premise at issue.
3. Place an x only in an area where it is possible for there to be an object.
4. If it is not clear where an x is to be placed, it should straddle the line connecting
two circles:

5. Never place an x on a portion of a line that does not relate two and only two
circles.

TOPIC 3: DIAGRAMMING IN THE TRADITIONAL INTERPRETATION

The only difference between diagramming a categorical syllogism in the traditional


interpretation and diagramming a categorical syllogism in the modern interpretation is
that since the former assumes existential import, any diagram of a universal proposition
will include an x:

40
We can tell from the shading and encircled x’s that we have a universal affirmative (“All
M are P“) and a universal negative (“No M are S,“ or perhaps “No S are M“).

TOPIC 4: MOOD AND FIGURE

In categorical syllogisms, the S term (the subject term of the conclusion) can be either
the subject or the predicate of the premiss in which it occurs; similarly, the P term (the
predicate term of the conclusion) can be either the subject or the predicate of the
premiss in which it occurs.

We know that each categorical syllogism contains a major premise, a minor premise,
and a conclusion. We also know that there is a middle term in each of the premises.
When the argument is in standard-form, this middle term can appear in four possible
ways, reflecting the figure of the syllogism:

MP PM MP PM

SM SM MS MS

SP SP SP SP

We also know that each proposition in a categorical syllogism in standard-form will have
a specific quantity. The premises and conclusion can all be A-propositions, for example.
Since there are 256 permutations of the categorical syllogism, each reflecting a
different figure, which is the proposition type for each of the standard-form categorical
syllogism’s quantity possibilities.

Together, the mood and figure tell us everything we need to know in order to test a
standard-form categorical syllogism for validity.

Mood and Figure. Mood and figure give you two important pieces of information about
a categorical syllogism. Mood tells you the quantity and quality of the major premise,
the minor premise, and the conclusion of a categorical syllogism. Figure tells you the
placement of the middle term in the premises.

TOPIC 5: RULES AND FALLACIES

There are six rules a standard-form categorical syllogism must meet in order to be valid.
If it fails to meet any one of these rules, it is invalid. Each rule has an accompanying
fallacy that alerts us to the specific way in which a categorical syllogism can be invalid.

1. The middle term must be distributed at least once.


2. If a term is distributed in the conclusion, it must also be distributed in its
corresponding premise.
3. A categorical syllogism cannot have two negative premises.
4. A negative premise must have a negative conclusion.
5. A negative conclusion must have a negative premise.
6. Two universal premises cannot have a particular conclusion.

Rule 1: We understand the concept of distribution, from Chapter 5, as referring to the


entirety of a class being included in, or excluded from, another class. We know that

41
universal propositions distribute the subject term, while negative propositions distribute
the predicate term. When the middle term is not distributed in either premise, the
argument commits the fallacy of undistributed middle.

Rule 2: A term does not have to be distributed in the conclusion, but if it is, find its
corresponding premise. The corresponding premise is the one that contains the term or
terms (minor or major—or both) distributed in the conclusion. For example, if the
conclusion is an E-proposition, both the subject and predicate terms are distributed.
These terms must also be distributed in their respective premise in order for the rule to
be met. If the rule is broken, the argument commits the fallacy of illicit major or the
fallacy of illicit minor.

Rule 3: If the premises are both negative, there is no way to establish a relation between
the major and minor terms. When this rule is broken, the argument commits the fallacy
of exclusive premises.

Rule 4: Rule 3 tells us that we cannot have two negative premises and still have a valid
argument. This does not mean, however, that we can never have a negative premise.
The only requirement in this case is that the conclusion also be negative, so that the
relation between major and minor terms can be established. When this rule is broken,
the argument commits the fallacy of affirmative conclusion/negative premise.

Rule 5: This rule is the converse of Rule 4. Rule 3 tells us that we cannot have two
negative premises and still have a valid argument. This does not mean, however, that
we can never have a negative conclusion The only requirement in this case is that one
premise also be negative, so that the relation between major and minor terms can be
established. When this rule is broken, the argument commits the fallacy of negative
conclusion/affirmative premise.

Rule 6: Recall that, in the modern interpretation, a universal categorical proposition


does not assume a member of the subject class exists. Therefore, if a categorical
syllogism’s conclusion is particular, we have no way of establishing the member of the
subject class. An argument that breaks this rule commits the existential fallacy. (Note: In
the traditional interpretation, the argument is provisionally valid. If the subject class in
the conclusion does not denote actually existing objects, the syllogism is invalid.)

TOPIC 6: ORDINARY LANGUAGE ARGUMENTS

Ordinary language arguments can be analyzed either by Venn diagram or the rules of
the syllogism. First, however, several guidelines must be followed:
1. If there are more than three classes of objects (three terms) in the argument,
the terms must be reduced.
2. Eliminate superfluous words to reveal the categorical structure, quantity, and
quality of the argument.
3. Identify synonyms and replace them with the terms appearing elsewhere in
the argument.
4. Use conversion, obversion, and contraposition to begin the process of
rewriting the argument in standard-form.
5. Eliminate prefixes as needed.

TOPIC 7: ENTHYMEMES

42
We saw in Section F that some categorical arguments contain too many terms. There
are also arguments, called enthymemes, which are incomplete. That is, the argument
may contain only one premise and a conclusion, or only two premises, and so forth. In
these cases, the goal is to make the argument complete, so that it can be rewritten as
necessary in standard-form.

TOPIC 8: SORITES

Still another type of incomplete argument (enthymeme) is the sorites. This is a chain* of
premises that lack intermediate conclusions. Here again, the goal is to establish a
complete categorical syllogism that can be tested for validity. If any syllogism in the
chain is invalid, the sorites is invalid.

The first step in the process is to rewrite the argument so that the premises appear one
on top of another, with a line demarcating the chain of premises from the conclusion:
Premise1
Premise2
Premise3
Conclusion

The first two premises are used to yield an intermediate conclusion, which then
becomes a premise in the next sequence:
Premise1
Premise2
Intermediate Conclusion
Premise3
Conclusion

*We say the reasoning forms a chain because there are repeated terms that link two
premises together.

Sorites
Both Chapter 3 and section F of this chapter cover incomplete arguments. If you’ve
understood the basics of argument diagramming, and you understand how to identify
what’s missing in an argument, you can successfully tackle the special type of
enthymeme called a sorites.

First, remember that a categorical syllogism has two premises and a conclusion. A
sorites has more than one premise and a conclusion. Your job is to organize the
argument so that you can create a series of categorical syllogisms out of a sorites.
1) Be sure the argument is translated into standard-form categorical propositions,
paraphrasing as appropriate.
2) Reveal the form of the argument by replacing terms with letters.
3) Put the argument elements in order by identifying the conclusion’s predicate. Then
locate that same term in one of the premises. This becomes the first premise. Repeat
until you’ve accounted for all of the premises. The subject term of the conclusion will be
found in the last premise of the sorites.
4) The first two premises should yield a conclusion that is presently missing. Assert that
conclusion.
5) You are now ready to diagram the first two premises. If the argument is valid, you can
use the conclusion as the first premise in the next leg of the argument.

43
ASSESSMENT

Name:__________________________________________ Date:_______________
Year & Section:___________________________________ Rating:______________

Direction: Encircle the letter of the correct answer.

Use Venn diagrams to determine whether the following categorical syllogism form
is valid or invalid under the modern (Boolean) interpretation.

1.
No M are P.
Some S are M.
Some S are P.

a. Valid

b. Invalid

c. Valid

d. Invalid

2.
Some M are P.
Some S are M.
Some S are not P.

44
a. Invalid

b. Invalid

c. Invalid

d. Valid

Use Venn diagrams to determine whether the following categorical syllogism form
is valid, provisionally valid, or invalid under the traditional (Aristotelian) interpretation.

3.
All M are P
No M are S
Some S are not P

a. Valid
b. Provisionally Valid
c. Invalid

4.
No P are M
No M are S
Some S are not P

a. Valid
b. Provisionally Valid
c. Invalid

Determine the mood and figure of the following syllogisms

5.
No P are M
Some S are M
Some S are not P

a. EIO-3
b. EIO-2

45
c. IEO-3
d. IEO-2

6.
All M are P
All S are M
All S are P

a. AAA-1
b. AAA-2
c. AAA-3
d. AAA-4

Use the tools discussed in this section to reduce the number of terms in the following
syllogisms. Then use the six rules to determine whether the syllogisms
are valid or invalid under the modern interpretation.

7.
All B are A
Some B are non-C
Some C are not A

a. Valid
b. Invalid: Rule 2 and Rule 3 are broken.
c. Invalid: Rule 2 and Rule 4 are broken.
d. Invalid: Rule 2 and Rule 5 are broken.
e. Invalid: Rule 2 and Rule 6 are broken.

8.
All C are B
All B are A
All A are non-C

a. Valid
b. Invalid: Rule 1 is broken.
c. Invalid: Rule 2 is broken.
d. nvalid: Rule 4 is broken.
e. Invalid: Rule 5 is broken.

Determine which statement is missing from the following argument, and whether or not
that argument is valid.

9. While it’s true that all smart people wear glasses, it’s definitely not the case that
tall people wear glasses.

a. Missing premise: No smart people are tall. Valid


b. Missing conclusion: No smart people are tall. Valid
c. Missing premise: No smart people are tall. Invalid
d. Missing conclusion: No smart people are tall. Invalid

10. Every year Global greenhouse gas emissions’ rise is detrimental to Earth’s
survival because every year more and more people eating meat is detrimental to
Earth’s survival.

46
a. Missing premise: All years more and more people eat meat are years
when greenhouse gas emissions rise. Valid
b. Missing conclusion: All years more and more people eat meat are
years when greenhouse gas emissions rise. Valid
c. Missing premise: All years more and more people eat meat are years
when greenhouse gas emissions rise. Invalid
d. Missing conclusion: All years more and more people eat meat are
years when greenhouse gas emissions rise. Invalid

LESSON VII
PROPOSITIONAL LOGIC
TOPICS
1. Logical Operators and Translations
2. Compound Statements
3. Truth Functions
4. Truth Tables for Propositions
5. Contingent and Noncontingent Statements
6. Logical Equivalence
7. Contradictory, Consistent, and Inconsistent Statements
8. Truth Tables for Arguments
9. Indirect Truth Tables

LEARNING OUTCOMES
At the end of the lesson, student should be able to:
 Expose student to the nature, types of propositions and their role for
preparing judgment.
 Enable students to create logical relations using opposing propositions.
 Orient students to the different criteria for truth and allow them to
evaluate their veracity.

Propositional logic is the study of how simple statements (basic components of


propositional logic) are modified to form compound statements and the way in which
the truth is a function of simple statements and compounding elements. A simple
statement is one that has no other statement as a component. A compound statement
is one which has at least one simple statement as a component The basic components
of propositional logic are statements.

TOPIC 1: LOGICAL OPERATORS AND TRANSLATIONS

47
There are two types of statement: simple and compound. A simple statement is one
that does not contain another statement as a component. Another way to describe a
simple statement is to say that it contains a subject and a verb. It can also contain
dependent clauses, but the basic idea is that the smallest grammatical unit has a truth
value. In the notation of symbolic logic, these statements are represented by capital
letters A–Z.

A compound statement contains at least one simple statement as a component, along


with a connective. There are five connectives: negation, conjunction, disjunction,
conditional, and biconditional. In the notation of symbolic logic, these connectives are
represented by operators:

Ordinary language is translated into symbolic logic notation using the aforementioned
capital letters A–Z. In general we translate connectives as follows:

When translating complex statements from ordinary language into symbolic logic
notation, isolate the simple sentences and the connectives.

TOPIC 2: COMPOUND STATEMENTS

Here are four rules to help organize your thinking about translating complex statements:
1. Remember that every operator except the negation is always placed in
between statements.
2. Tilde is always placed to the left of whatever is negated.
3. Tilde never goes, by itself, in between two statements.
4. “Parentheses, brackets, and braces are required in order to eliminate
ambiguity in a complex statement”. Consider any type of parenthetical a means
of grouping together as a single unit a complex statement as part of a larger
complex statement.

Every compound statement consisting of simple statements and two or more operators
is governed or driven by a main operator. There is always only one main operator in any
compound statement, and that operator is either one of the four that appear in
between statements or the tilde that appears in front of the statement that is negated.

TOPIC 3: TRUTH FUNCTIONS

We know that a statement has truth values: it is either true or false:

48
A compound statement also has truth values. These are functions of the simple
statements in combination with the meaning of a given operator:

Negation tells us, “It is not the case that…”

Conjunction tells us, “Both…are the case.”

Example:

If p, then q If I spend the weekend partying, then I will fail some midterms.
If r, then s If I spend the weekend studying, then I will miss out on some good times.
p or r I will party this weekend, or I will study.
q or s I will fail some midterms, or I will miss out on some good times.

Disjunction tells us that, “At least one is the case…”

Conditional mirrors the concept of validity: If the premises are true, the conclusion
cannot be false. When the premises are true and the conclusion is false, the inference is
invalid.

49
The biconditional tells us that, “Either both are the case, or neither is…”

TOPIC 4: TRUTH TABLES FOR PROPOSITIONS

When we construct a truth table to determine the possible truth values of a given
statement, we first need to know at least two things:

1. The number of simple statements that are in the compound statement. The
number of rows of the truth table follows this formula: R = 2number of simple statements.
2. The main operator. Any statement containing two or more operators will be
calculated according to an order of operations. The main operator determines
the final truth values of the statement, and as such its values are calculated last.

When truth values are provided, we calculate to the main operator using the same
order of operations as when we are calculating all possible truth values. If you are told
that P, Q, and R have values T, F, and T, respectively, you can calculate the value of the
compound statement, P ⊃ (Q • R):

TOPIC 5: CONTINGENT AND NONCONTINGENT STATEMENTS

A contingent statement is true* on at least one row of the truth table, and false on at
least one row of the truth table. In other words, such a statement is sometimes true and
sometimes false. A noncontingent statement is not dependent on the truth values of
the component parts. Such a statement is either always true (a tautology) or always
false (self-contradictory).

TOPIC 6: LOGICAL EQUIVALENCE

Two or more statements are logically equivalent when they have the same values* on
each row of the truth table. We will see in Chapter 8 that the replacement rules are
based on such equivalences. Truth tables for the following pairs of statements will
reveal their logical equivalence. They are just a few of myriad truth-functionally
equivalent statements:

P ⊃ Q and ~ P v Q
P ⊃ Q and ~ Q ⊃ ~ P
~ (P • Q) and ~ P v ~ Q

50
TOPIC 7: CONTRADICTORY, CONSISTENT, AND INCONSISTENT STATEMENTS

Two statements are contradictory when they are never true or never false* at the same
time on the same row of the truth table.

Two or more statements are consistent when they are true* at the same time on at
least one row of the truth table.

Two or more statements are inconsistent when they are not true* at the same time on
even one row of the truth table.

*Under the main operator.

TOPIC 8: TRUTH TABLES FOR ARGUMENTS

We have seen that truth tables can be constructed for individual statements and pairs of
statements. The truth table allows us to classify statements according to their truth
values under the main operator. The same can be done for arguments in order to
determine validity or invalidity.

A truth table in which there is even one row of the argument on which the premises are
all true while the conclusion is false is invalid. A truth table in which there is not even
one row of the argument on which the premises are all true while the conclusion is false
is valid.

TOPIC 9: INDIRECT TRUTH TABLES

The indirect truth table is a powerful and expedient tool for testing an argument’s
validity:

1. Assume the premises are true and the conclusion is false. In other words,
assume the argument is invalid. This involves placing a T under the main
operator of each of the premises, and an F under the main operator of the
conclusion. If any of the statements is simple, the T or the F is placed directly
beneath it.
2. Based on the truth definitions for each of the operators, calculate backward
from the assumed value to the statement’s constituent elements.
3. If you can calculate to the constituent elements without contradicting the truth
definitions for the given operator, the argument is invalid.
4. If you cannot calculate to the constituent elements without contradicting the
truth definitions for the given operator, the argument is valid.

Explore

Truth Functions. The truth value of a complex statement is a function of the meaning of
the operator and the truth values of the simple statements that, along with the
operator, make it up.

51
 Tilde is false when and only when the statement it negates is true.
Tilde is true when the statement it negates is false.
 Dot is true when and only when both sides of dot are true;
otherwise, it is false.
 Wedge is false when and only when both sides of wedge are false;
otherwise it is true.
 Horseshoe is false when and only when the antecedent is true and
the consequent is false; otherwise, it is true.
 Triple bar is true when and only when the truth values of each
side are the same.

Truth Tables for Propositions. Work from the inside out: begin with the truth values of
the simple statements, then determine the truth values of the least complicated
compound statement, all the way out to the main operator. For example, the truth
value of the statement, ~ [A ⊃ ~ (B v C)] is worked out as follows:
1) Determine the truth value of “B v C” according to the truth definitions of “v”.
2) Determine the truth value of “~ (B v C)” according to the truth definitions of
“~”. Notice that in this statement, “~” is a denial of “v”.
3) Determine the truth value of “A ⊃ ~ (B v C)” according to the truth definitions
of “⊃ ”. Notice that “⊃ ” is determined by comparing the values of the
antecedent, “A” with the values of the consequent, “~”.
4) determine the value of the main operator, ~, by negating the value of the “ ”
within the brackets.

Remember, when determining the truth value of the statement, to work from the
“inside, out.” Begin with the possible values for each simple statement. Next, work out
the value for the least compound statement, and so on until you reach the main
operator. The truth value you determine last is the one for the main operator. For
example, here’s the order of operations for the following statement:

Indirect Truth Tables. The indirect proof method of testing for validity nicely reveals the
principle of non-contradiction at work in a valid argument. When we assume the
premises are true and the conclusion is false, but the argument is valid, we run into a
contradiction:

Assume the following:

Assuming that “~ A” is false, then “A” is true. Now since we assumed that “A” is
true, the wedge is true because at least one of the disjuncts is true. In this case, ~
(A v B) is false. In other words, you can’t have “v” be true and ~ (A v B) true at
the same time.

If the conclusion is false, then the premise cannot be true:

52
ASSESSMENT

Name:__________________________________________ Date:_______________
Year & Section:___________________________________ Rating:______________

Direction: Choose the correct translation of the following statements. Encircle the letter
of the correct answer.

1. You will pass the class only if you study.

Let P = you pass the class and S = you study.

a. ~ S ⊃ P
b. S ⊃ P
c. P ⊃ S
d. P v ~ S

2. I will eat either tofu or steak for dinner but certainly not both of them.

Let T = I eat tofu for dinner and S = I eat steak for dinner

a. T v S
b. T • S
c. T v S • (~ T • ~ S)
d. ( T v S ) • ~ ( T • S )

3. I will eat either tofu or steak for dinner.

Let T = I eat tofu for dinner and S = I eat steak for dinner

a. T
b. S
c. T v S
d. T • S
53
4. I lost my cat, but my dog is still here.

Let C = I lost my cat and D = my dog is still here

a. C
b. D
c. C v D
d. C • D

Translate the following statements into symbolic form by using logical operators and
uppercase letters to represent the English statements.

5. If you have neither a bad credit score nor debt, then you can buy a house.

Let B = you have a bad credit score, D = you have debt, and H = you can buy a
house.

a. ( B v D) ⊃ H
b. (~ B v ~ D ) ⊃ H
c. ~ [ ( B ν D ) ⊃ H ]
d. ~ ( B v D ) ⊃ H

6. If you have a good credit score and no debt, then you can buy a house.

Let G = you have a good credit score, D = you have debt, and H = you can buy a
house.

a. G • D • H
b. ( G • D ) ≡ H
c. ( G • ~ D ) ≡ H
d. ( G • ~ D ) ⊃ H

7. If “X • ~ Y” is true, then which of the following is correct?

a. Y must be true.
b. Y must be false.
c. Y could be true or false.
d. X must be false.

8. If “X • Y” is true, then which of the following is correct?

a. X must be true.
b. X must be false.
c. Both X and Y must be true.
d. Both X and Y must be false.
e. Y could be true or false.

Determine whether the truth table is correct for the following compound propositions.

9.

54
a. Correct
b. Incorrect

10. Q ⊃ P

a. Correct
b. Incorrect

Create a truth table to determine whether the following statement is contingent,


a tautology, or a self-contradiction.

11. R v ( S • ~ S )

a. The statement is contingent.


b. The statement is a tautology.
c. The statement is a self-contradiction.

12. Q • Q

a. The statement is contingent.


b. The statement is a tautology.
c. The statement is a self-contradiction.

Create a truth table to determine whether or not the following statements are logically
equivalent.

13. ~ ( P • Q ) | ~ P v ~ Q

a. The statements are logically equivalent.


b. The statements are not logically equivalent.

55
14. ( P v Q ) | ~ P • ~ Q

a. The statements are logically equivalent.


b. The statements are not logically equivalent.

Create a truth table to determine whether the following statements


are contradictory, consistent, or inconsistent.

15. P ⊃ Q | P • ~ Q

a. Contradictory
b. Consistent
c. Inconsistent

16. ~ P • Q | ~ Q v P

a. Contradictory
b. Consistent
c. Inconsistent

Create a truth table to determine whether the following arguments are valid or invalid.

17. ~ P v ~ S
P /S

a. Valid
b. Invalid

18. P v Q
~Q /~P

a. Valid
b. Invalid

First, translate the following arguments using the logical operators. Second, create a
truth table to determine whether the arguments are valid or invalid.

19. If Tom and Kait got married yesterday, they are on their honeymoon today. I
was at their wedding yesterday, when they got married, so they must be
honeymooning today.

a. Valid
b. Invalid

Construct an indirect truth table to determine whether or not the following arguments
are valid.

56
20. ( P • Q ) v R
Q
~P /R

a. Valid
b. Invalid

LESSON VIII
NATURAL DEDUCTION
TOPICS
1. Implication Rules I
2. Tactics and Strategy
3. Implication Rules II
4. Replacement Rules I & F.II
5. Conditional and H. Indirect Proof

LEARNING OUTCOMES
At the end of the lesson, student should be able to:
 Initiate student to think critically by applying the rules of correct thinking.

Natural deduction is a proof procedure by which the conclusion of an argument is


validly derived from the premises through the use of rules of inference. The function of
rules of inference is to justify the steps of a proof. A proof is a sequence of steps in
which each step is either a premise or follows from earlier steps in the sequence
according to the rules of inference. There are two types of rules of inference:
implication rules and replacement rules. Implication rules are valid argument forms that
are validly applied only to an entire line. Replacement rules are pairs of logically
equivalent statement forms (they have identical truth tables) that may replace each
other within the context of a proof. Unlike the eight implication rules that are restricted
to entire lines of a proof, replacement rules have no such restriction. They can be used
either for an entire line or a part of a line.

Deriving individual lines in a proof involves tactics, or the use of small-scale maneuvers
or devices. You may look at one or two lines in a proof to determine which move to
make next. When deciding how to approach an argument, you develop a strategy, which

57
is the plan to achieve the overall goal. In a proof, the overall goal is to derive the
conclusion of a valid argument.

For certain types of argument, using subproof strategies is most efficacious. Two
common subproofs are conditional proof (CP) and indirect proof (IP). CP is a method
that starts by assuming the antecedent of a conditional statement on a separate line
and then proceeds to validly derive the consequent on a separate line. When the result
of a conditional proof sequence is discharged it no longer needs to be indented. IP is a
method that starts by assuming the negation of the required statement and then validly
deriving a contradiction on a subsequent line

Natural Deduction. The system of natural deduction is a specific proof procedure based
on the truth definitions of the logical operators, ~, •v, ⊃, and ≡. This system
uses implication rules, which are valid argument forms, to justify each step in the
derivation of a valid argument’s conclusion. This system also uses replacement rules,
which are pairs of logically equivalent statements, to aid in the derivation of a valid
argument’s conclusion.

TOPIC 1: IMPLICATION RULES I

Modus ponens (MP), modus tollens (MT), hypothetical syllogism (HS), and disjunctive
syllogism (DS) make up the first four implication rules in this system of natural
deduction:

MP
P⊃Q
P
Q

MT
P⊃Q
~Q
~P

HS
P⊃Q
Q⊃R
P⊃R

DS
PvQ
~P
Q

Each implication rule involves dismantling an operator (horseshoe and wedge). In the
case of HS, the horseshoe’s elements are rearranged. It is also worth noting that each of
these rules involves drawing an inference from two statements.

58
TOPIC 2: TACTICS AND STRATEGY

Strategy is the overall approach you take to setting up a proof. This will be increasingly
important to think about when the arguments become more complex. Tactics are small-
scale maneuvers—they are currently what you’re using to work your way through
proofs at this point in your skill set. In other words, rules like MP and DS are tactics.

The following strategies will help you form the foundation for solving more complicated
proofs later on:

1. Simplify and isolate using the following tactics: MP, MT, and DS.
2. Look for negation using the following tactics: MT and DS.
3. Look for conditionals using the following tactics: MP, MT, HS.
4. Look at the conclusion and work backward using the following tactic: anticipate
what you need to reach the conclusion by looking for the line that should
immediately precede it.

TOPIC 3: IMPLICATION RULES II

Constructive dilemma (CD), simplification (Simp), conjunction (Conj), and addition (Add)
make up the second four implication rules in this system of natural deduction:

CD
(P ⊃ Q) • (R ⊃ S)
PvR
QvS

Simp
P•Q
P

Conj
P
Q
P•Q

ADD
P
PvQ

As with the first four implication rules, the second four involve assembling or
disassembling statements according to their main operators’ truth definitions.
The following strategies will help you form the foundation for solving more complicated
proofs later on:
1. Simplify and isolate using the following tactics: MP, MT, DS, and Simp.
2. Look for negation using the following tactics: MT and DS.
3. Look for conditionals using the following tactics: MP, MT, HS, and CD.

59
4. Look at the conclusion and work backward using the following tactic:
anticipate what you need to reach the conclusion by looking for the line that
should immediately precede it.
5. Add whatever you need using the following tactic: Add.
6. Combine lines by using the following tactic: Conj.

TOPIC 4: REPLACEMENT RULES I

Both sets of replacements rules (Sections E and F) enlist the principle of replacement,
which states that logically equivalent expressions may replace each other within the
context of a proof. Replacement rules can be used either on a part of a line or an entire
line in a proof.

DeMorgan (DM), commutation (Com), association (Assoc), distribution (Dist), and


double negation (DN) are sets of equivalent statements, one of which can be substituted
for the other in a proof. Replacement rules aid you in assembling or disassembling
statements.

The following strategies will help you form the foundation for solving more complicated
proofs later on:

1. Simplify and isolate using the following tactics: MP, MT, DS, and Simp.
2. Look for negation using the following tactics: MT, DS, DM, and DN.
3. Look for conditionals using the following tactics: MP, MT, HS, and CD.
4. Look at the conclusion and work backward using the following tactic: anticipate
what you need to reach the conclusion by looking for the line that should
immediately precede it.
5. Add whatever you need using the following tactic: Add.
6. Combine lines by using the following tactic: Conj.
7. Regroup lines by using the following tactics: Com, Assoc, Dist.

TOPIC 5: REPLACEMENT RULES II

Transposition (Trans), material implication (Impl), material equivalence (Eqiv),


exportation (Exp), and tautology (Taut) make up the last set of replacement rules.

TOPIC 6: CONDITIONAL PROOF

Conditional Proof allows you to prove a horseshoe statement by assuming its


antecedent and deriving its consequent. It’s a veridical proof that says, “If I
assume P and derive from it Q, then I can say,‘P ⊃ Q.’ ”

Conditional proof is a good strategy to use for setting up the framework of a proof,
particularly if the conclusion of the argument is a conditional claim. As with all
assumptive proofs, the assumption must be “discharged,” which is to say, you must get
out of the subproof sequence and back onto the main line. This is accomplished by
achieving the goal of the subproof.

TOPIC 7: INDIRECT PROOF

60
Indirect Proof works similarly to an indirect truth table. The basic strategy is to assume
the conclusion is false. From it a contradiction is derived, which allows you to assert that
the assumption is false, which is to say, the conclusion is true. Of course, you needn’t
construct indirect proof exclusively with the negated conclusion as the assumptive line.
There may be times in a proof when you need a negated statement, and the only way to
derive it is with indirect proof.

As with all assumptive proofs, the assumption must be “discharged,” which is to say,
you must get out of the subproof sequence and back onto the main line. This is
accomplished by achieving the goal of the subproof.

Explore

Implication Rules I
 Think of the first four rules of inference as rules allowing you to create complex
statements (hypothetical syllogism) or break them apart (modus ponens, modus
tollens, and disjunctive syllogism).

Implication Rules II
 The second four inference rules allow you to create complex statements
(conjunction and addition), or break them apart (constructive dilemma and
simplification).
 Don’t be intimidated by the complexity of constructive dilemma. It’s basically
two instances of modus ponens:

Separate out the left side of dot in premise 1, the left side of wedge in premise 2,
and the left side of wedge in the conclusion:
p⊃q
p
q

Now do the same for the right side of the main operator in premise 1, the right
side of wedge in premise 2, and the right side of wedge in the conclusion:
r⊃s
r
s

In both cases, you’ve got modus ponens! Now, if it’s true that you have
both p ⊃ q and r ⊃ s, and you have p or r, it follows that you have q or s.

Replacement Rules I and 8F Replacement Rules II


 Think about DeMorgan’s as instances of “pushing the negation through,” and
swapping out wedge for dot, or dot for wedge.
 Remember, logically equivalent statements are those that mean logically the
same thing. So, for example, to assert one statement, p • q is to assert its
logically equivalent statement q • p.
 Association involves moving a pair of parentheses from one conjunction in a
complex sentence to another conjunction in the same sentence. The same
applies to a sentence involving more than one disjunction.
 If you’re uncertain about whether a replacement rule really reflects logical
equivalence (and some may be counterintuitive), construct a truth table. Use
your knowledge of logical equivalence from Chapter 7F.

61
ASSESSMENT

Name:__________________________________________ Date:_______________
Year & Section:___________________________________ Rating:______________

Direction: Encircle the letter of the correct answer.

Use the first four implication rules to determine whether or not the proofs for the
following arguments are correct.

1. 1. J ~ K
2. K ( L ⊃ J )
3. ~ J / L⊃J
The following proof is correct:
1. J ~ K
2. K ( L ⊃ J )
3. ~ J / L⊃J
4. ~ K 1,3 DS
5. L ⊃ J 2,4 DS
a. True
b. False
2. 1. P ⊃ Q
2. Q ⊃ R
3. R ⊃ S /P⊃S
The following proof is correct:
1. P ⊃ Q
2. Q ⊃ R
3. R ⊃ S /P⊃S
4. Q ⊃ P 2,3, HS
5. P ⊃ S 1,4, HS

62
a. True
b. False
3. 1. Q ⊃ ( R v S )
2. ~ ( R v S ) /~Q
The following proof is correct:
1. Q ⊃ ( R v S )
2. ~ ( R v S )
3. ~ Q 1,2, MT
a. True
b. False

Use the eight implication rules to determine whether or not the proof of the following
arguments are correct.

4. 1. ( P v Q ) ⊃ ~ R
2. R v S
3. P /S

The following proof is correct:


1. ( P v Q ) ⊃ ~ R
2. R v S
3. P /S
4. P v Q 3, Conj
5. ~ R 1,4, MP
6. S 2,5, DS
a. True
b. False
5. 1. ( P ⊃ S ) • ( R ⊃ Q )
2. P v R
3. ( S ⊃ R ) • ( Q ⊃ T ) /R vT

The following proof is correct:


1. ( P ⊃ S ) • ( R ⊃ Q )
2. P v R
3. ( S ⊃ R ) • ( Q ⊃ T ) /R vT
4. S • Q 1,2, CD
5. R v T 3,4, CD
a. True
b. False
6. 1. ( N ⊃ O ) • ( P ⊃ Q )
2. ( R ⊃ S ) • ( S ⊃ T )
3. N v R /OvS

The following proof is correct:


1. ( N ⊃ O ) • ( P ⊃ Q )
2. ( R ⊃ S ) • ( S ⊃ T )
3. N v R /OvS

63
4. N ⊃ O 1, Simp
5. R ⊃ S 2, Simp
6. ( N ⊃ O ) • ( R ⊃ S ) 4,5, Conj
7. O v S 3,6, CD
a. True
b. False

Determine whether or not the proof for the following arguments are correct.

7. 1. ( R ⊃ S ) • ( P ⊃ Q )
2. ~ (~ P • ~ R )
3. ~ Q /S

The following proof is correct:


1. ( R ⊃ S ) • ( P ⊃ Q )
2. ~ (~ P • ~ R )
3. ~ Q /s
4. ~ ( P v R ) 2, DM
5. P v R 4, DN
6. ( P ⊃ Q ) • ( R ⊃ S ) 1, Com
7. Q v S 5,6 CD
8. S 3,7 DS
a. True
b. False

8. 1. ~ ( R v S ) ⊃ ( P v Q )
2. ~ R • ~ S / PvQ
The following proof is correct:
1. ~ ( R v S ) ⊃ ( P v Q )
2. ~ R • ~ S / PvQ
3. ~ ( R v S ) 2, DM
4. P v Q 1,4, MP
a. True
b. False

9. 1. ~ P v ( Q v ~ R )
2. ~ Q /~(P•R)

The following proof is correct:


1. ~ P v ( Q v ~ R )
2. ~ Q /~(P•R)
3. (~ P v Q ) v ~ R 1, Assoc
4. ( Q v ~ P v ~ R ) 3, Com
5. Q v (~ P v ~ R ) 4, Assoc
6. ~ P v ~ R 2, 5, DS
7. ~ ( P • R ) 6, DM

64
a. True
b. False

10. 1. ( P v Q ) v R
2. ~ P /QvS

The following proof is correct:


1. ( P v Q ) v R
2. ~ P /QvS
3. P v ( Q v R ) 1, Impl
4. Q v R 2,3,DS
a. True
b. False

LESSON IX
PREDICATE LOGIC

TOPICS
1. Translating Ordinary Language
2. Four New Rules of Inference
3. Change of Quantifier
4. Conditional Indirect Proof
5. Demonstrating invalidity
6. Relational Predicates,
7. Identity

LEARNING OUTCOMES
At the end of the lesson, student should be able to:
 To identify structural similarities between domains and further discovery to
form conclusion.

Predicate logic integrates many of the features of categorical and propositional logic. It
combines the symbols associated with propositional logic with special symbols that are
used to translate predicates. Predicates are the fundamental units in predicate logic.
Uppercase letters, called “predicate symbols,” are used to symbolize these units. The
subject of a singular statement (a statement about an individual) is translated using
lowercase letters. The lowercase letters, called “individual constants,” act as names of
individuals.

There are two types of statements in predicate logic: singular and quantified. Quantified
statements are statements about an entire class or at least one in a class. The universal
quantifier is the symbol that is used to capture the idea that universal statements assert
something about every member of the subject class. The existential quantifier is used to
capture the idea that existential statements assert something about at least one
member of the subject class.

65
The three lowercase letters x, y, and z, are individual variables. These are used as
placeholders for things quantified universally or existentially. The universal quantifier is
formed by putting a lowercase letter x, y, or z in parentheses. The existential quantifier
is formed by putting a backward E in front of a variable, and then placing them both in
parentheses.

Universally and existentially quantified statements reflect a domain of discourse,


namely, the set of individuals over which a quantifier ranges. Variables governed by a
quantifier are called bound variables. When there is no quantifier, the expression is not
a statement, but a statement function. That is, when there is no quantifier, but there is
a predicate and a variable, the expression does not make any universal or particular
assertion about anything; therefore, it has no truth value. Statement functions are
simply patterns for a statement. Variables in a statement function are “free,” which is to
say, they are not governed by any quantifier.

It is important to be conversant with the following concepts:

 When instantiation is applied to a quantified statement, the quantifier is


removed, and every variable that was bound by the quantifier is replaced by the
same instantial letter.
 Universal instantiation (UI): The rule by which we can validly deduce the
substitution instance of a statement function from a universally quantified
statement.
 Universal generalization (UG): The rule by which we can validly deduce the
universal quantification of a statement function from a substitution instance
with respect to the name of any arbitrarily selected individual (subject to
restrictions).
 Existential generalization (EG): The rule that permits the valid introduction of an
existential quantifier from either a constant or a variable.
 Existential instantiation (EI): The rule that permits giving a name to a thing that
exists. The name can then be represented by a constant.
 The four new rules of predicate logic are similar to the eight implication rules, in
that they can be applied only to an entire line of a proof (either a premise or a
derived line).
 Change of quantifier rule: Allows the removal or introduction of negation signs.
The rule is a set of four logical equivalences.
 Universal generalization cannot be used within an indented proof sequence if
the instantial variable is free in the first line of the sequence.
 A counterexample to an argument is a substitution instance of an argument form
that has true premises and a false conclusion.
 The finite universe method of demonstrating invalidity assumes a universe,
containing at least one individual, to show the possibility of true premises and a
false conclusion.
 Monadic predicate: A one-place predicate that assigns a characteristic to an
individual thing.
 Relational predicate: Establishes a connection between individuals.
 Symmetrical relationship: Can be illustrated by the following: If A is married to B,
then B is married to A.
 Asymmetrical relationship: Can be illustrated by the following. If A is the father
of B, then B is not the father of A.
 Nonsymmetrical: When a relationship is neither symmetrical nor asymmetrical.
For example: If Kris loves Morgan, then Morgan may or may not love Kris.
 Transitive relationship: Can be illustrated by the following: If A is taller than B,
and B is taller than C, then A is taller than C.

66
 Intransitive relationship: Can be illustrated by the following: If A is the mother of
B, and B is the mother of C, then A is not the mother of C.
 Nontransitive relationship: Can be illustrated by the following: If Kris loves
Morgan and Morgan loves Terry, then Kris may or may not love Terry.
 Identity relation: A binary relation that holds between a thing and itself.
 Definite description: Describes an individual person, place, or thing.
 Reflexive property: The idea that anything is identical to itself.
 Irreflexive relationship: Can be illustrated by the following expression: “Nothing
can be taller than itself.”
 Nonreflexive: When a relationship is neither reflexive nor irreflexive.

Predicate logic integrates the most powerful features of categorical and propositional
logics, thereby allowing for a more extended scope of argument analysis than either of
the two can achieve individually. Predicate logic analyzes and expresses arguments
involving individuals, classes of things, and their relations. The formal notation combines
propositional logic symbols with special predicate logic symbolization in such a way as to
allow for the formal expression of complex yet precise language.

TOPIC 1: TRANSLATING ORDINARY LANGUAGE

There are two types of statements in predicate logic: singular and quantified. “A
singular statement is about a specific person, place, time, or object”. A quantified
statement is about classes of things. Such statements are either universal or particular.
“Universal statements either affirm or deny that every member of a subject class is a
member of a predicate class”. “Particular statements either affirm or deny that at least
one member of a subject class is a member of a predicate class.

There are two elements in a singular statement: predicate and individual constant. The
predicate of a singular statement is the fundamental unit and is translated with a capital
letter, A–Z. The subject of a singular statement is called an individual constant and is
translated with a lowercase letter, a–w:

Ordinary Language Predicate Logic Translation


Joe is a dog Dj

Notice that x, y, and z are not individual constants. Lowercase letters, x, y, and z are
enlisted as placeholders (called individual variables) in quantified statements. That’s
because quantified statements do not specify things, only classes of things. Things are
included in, or excluded from, classes:

All dogs are mammals.


No dogs are cats.
Some dogs are beagles.
Some cats are not friendly animals.

Notice that quantifiers and classes are features of predicate logic borrowed from
categorical logic. What is borrowed from propositional logic are the logical operators, ~,
•, v, ⊃, ≡:
Statement
Ordinary Language
Function
All dogs are mammals. Dx ⊃ Mx
No dogs are cats. Dx ⊃ ~Cx

67
Some dogs are beagles. Dx • Bx
Some cats are not friendly
Cx • ~Fx
animals.

The statement functions are expressions that do not make any universal or particular
assertion about anything; therefore, they have no truth value. Their variables are free,

“All” and “no” are universal quantifiers. They are translated as follows: (x). “Some” is a
particular quantifier and is translated as follows: (∃x).
Predicate Logic
Ordinary Language
Translation
All dogs are mammals. (x)(Dx ⊃ Mx)
No dogs are cats. (x)(Dx ⊃ ~Cx)
Some dogs are beagles. (∃x)(Dx • Bx)
Some cats are not
(∃x)(Cx • ~Fx)
friendly animals.

Notice that the appearance of the quantifiers includes parentheses around what are
otherwise statement functions. These parentheses tell us the domain of discourse,
which is “the set of individuals over which a quantifier ranges”. This domain of discourse
tells us that the variables in the statement function are bound by the quantifier:

For any x, if x is a dog, then x is a mammal.


For any x, if x is a dog, then x is not a cat.
There is at least one x that is a dog and a beagle.
There is at least one x that is a cat and not a friendly animal.

TOPIC 2: FOUR NEW RULES OF INFERENCE

There are four quantifier rules of inference that allow you to remove or introduce a
quantifier:

Universal Instantiation: Consider what a universally quantified statement asserts,


namely that the entirety of the subject class is contained within the predicate class.
Therefore, any instance of a member in the subject class is also a member of the
predicate class.

All dogs are mammals.


Joe is a dog.
Therefore, Joe is a mammal.

(x)(Dx ⊃ Mx)
Dj
Mj

The only way MP can be employed is if we remove the universal quantifier, which, as we
saw from the explanation above, can be done by naming a member of the subject class
in the universally quantified statement:

(x)(Dx ⊃ Mx
Dj ⊃ Mj

68
In this case, we use the individual constant, j, because the statements need to match up
if we are to use MP. Notice also that the instantiation applies to the entire line. You may
not substitute constants.

Universal Generalization: Moving from a universally quantified to a singular statement


is not controversial. On the other hand, we can recognize pretty quickly that we can’t go
the other direction quite as easily. Consider the following statement: “Joe the dog is an
American Staffordshire terrier.” It wouldn’t feel right, intellectually, to infer from this
statement that all dogs are American Staffordshire terriers. So, when we want to make
an inference to a universal statement, we may not do so from an individual constant:

Not Permitted:
Sa
(x)Sx

Instead, the generalization must be made from a statement function, where the
variable, by definition, could be any entity in the relevant class of things:

Dx ⊃ Mx
(x)(Dx ⊃ Mx)

The following account is a bit inaccurate, insofar as a statement function, by definition


does not assert anything, but we can still get the idea of the sort of move involved in
UG. If it’s the case that entities x are members of the D class, then they’re also members
of the M class. This phrase, “entities x,” suggests the quantity is not limited.

Notice also that the generalization applies to the entire line. You may not substitute
variables.

Existential Generalization: Like UI, EG is a fairly straightforward inference. Take the


sentence “Joe is an American Staffordshire terrier dog.” The sentence specifies an
existing American Staffordshire terrier. So, if Joe is one, it follows that at least one
American Staffordshire terrier exists:

Aj
(∃x)(Dx • Ax)

Notice also that the generalization applies to the entire line. You may not substitute
variables.
Existential Instantiation: Just as we have to be careful about generalizing to universally
quantified statements, we also have to be careful about instantiating an existential
statement. Suppose you are standing outside a room and you assert, “Someone in the
room is wearing glasses.” Would you be entitled to infer from that statement that
Matilda is wearing glasses? No. The best you can do is say, “Well, let’s assume that
Matilda is wearing glasses.” Now, here’s the tricky part. When you instantiate an
existential statement, you cannot choose a name that is already in use. That’s because
we are not justified in assuming that the individual constant is the same from one
instantiation to another—and don’t forget, it’s called an individual constant. So, if you
have to instantiate a universal statement and an existential statement, instantiate the
existential first. The universal instantiation can then assert the same constant as the
existential instantiation, because there are no restrictions on UI.

TOPIC 3: CHANGE OF QUANTIFIER

69
You can infer existential statements from universal statements, and vice versa, without
having to instantiate first. In ordinary language, the phrase equivalences are as follows:

“All are” is equivalent to “It’s not the case that there is one that is not.”
“It is not the case that all are not” is equivalent to “Some are.”
“Not all are” is equivalent to “Some are not.”
“It is not the case that there is one” is equivalent to “None are.”

(x)Sx :: ~(∃x)~Sx
~(x) ~Sx :: (∃x)Sx
~(x)Sx :: (∃x) ~Sx
(x) ~Sx :: ~(∃x)Sx

TOPIC 4: CONDITIONAL AND INDIRECT PROOF

In predicate logic, conditional and indirect proof follow the same structure as in
propositional logic:

1) Assume a statement, P, derive another statement, Q,, then discharge the assumptive
proof by asserting P ⊃ Q.
2) Assume a statement, P, derive another statement, ~P, then discharge the assumptive
proof by asserting ~P.
In predicate logic, however, there is one restriction on UG in an assumptive proof: when
the assumption is a free variable, UG is not allowed from the line where the free
variable occurs. That is because the assumption names an individual assumed to have
the property designated by the predicate. It does not, therefore, act as an arbitrary
individual from which we may generalize to a universal statement.

TOPIC 5: DEMONSTRATING INVALIDITY

There are two methods to demonstrate that a predicate logic argument is


invalid: Counterexample method and finite universe method. The counterexample
method follows the same steps used in Chapter 6: replace the premises with another set
we know to be true; replace the conclusion with one we know to be false. Each
replacement must follow the same form as the original:

All oranges are fruits.


Some vegetables are not fruits.
Some oranges are not vegetables.

(x)(Ox ⊃ Fx)
(∃x)(Vx • ~Fx)
(∃x)(Ox • ~Vx)

All citizens are eligible to vote.


Some people are not eligible to vote.
Some citizens are not people.

The finite universe method enlists indirect truth tables to show, truth-functionally, that
a predicate logic argument is invalid:

70
1. Suppose a universe that contains only one member.
2. Instantiate the premises and conclusion to the same constant.
3. Construct an indirect truth table to determine whether or not the argument is
invalid.
4. If the argument does not prove invalid with a single-member universe, try two
members:
a. Universal statements are translated with conjunction, since a universal
statement is the sort of claim that asserts that each individual in the
subject class is a member of the predicate class. So, this member, a, and
this member, b will both be members of the predicate class.
b. Existential statements are translated with disjunction, since an existential
statement is the sort of claim that asserts at least one individual in the
subject class is a member of the predicate class. So, this member, a, or
this member, b, is a member of the predicate class.

Note: if you do not prove the argument is invalid assuming a three-member universe,
double-check your work and then consider using the inference rules to construct a
proof. It may be that the argument is, in fact, valid.

TOPIC 6: RELATIONAL PREDICATES

We mentioned earlier that one of the features of predicate logic that make for a more
powerful system than either categorical or propositional logic alone is its flexibility. The
notation allows us to notate and work with relational predicates (two or more place
predicates), rather than only single-place predicates:
Joe is to the right of Stew: Rjs
Joe and Stew like each other: Ljs • Lsj
Everyone likes someone: (x)(Px (y)Lxy)

Proofs generally work the same way with relational predicates as they do with single-
place predicates. The same restrictions are also in place, with two additions:

1) UG cannot be used if the instantial variable is free on any line obtained by EI.
2) UI cannot be used if the free variable is already bound by an existential quantifier.

TOPIC 7: IDENTITY

One final way in which predicate logic is more robust than either propositional or
categorical logic is its ability to deal with identity statements. Identity is a two-way
relation holding between a thing and itself. A simple example makes this definition
clear: “50 Cent is Curtis Jackson.”

The symbolic notation for identity statements is the use of =. So, “50 Cent is Curtis
Jackson,” becomes f = c. When we deny identity, we use ≠. So, “50 Cent is not Marshall
Mather,” becomes f ≠ m.

When we want to distinguish between members of a class, but the statement we assert
does not specify names, we can use the identity symbol to help. “At least two dogs are
in the yard,” becomes (∃x)( ∃y)(Dx • Dy • Px • Py • x ≠ y). Similarly, when we want to
assert an exact number, but we do not specify names, we use the identity symbol.
“There is exactly one dog playing in the yard,” becomes (∃x)(Dx • Px • (y)[(Dy • Py) ⊃ x =
y).
71
Explore

Translating Ordinary Language


 Categorical and truth-functional statements combine to form the statements of
predicate logic:
o Individuals (the grammatical subject of the statement) are denoted with
lower case letters, a–z.
o What’s predicated of individuals (the grammatical predicate of the
statement) are denoted with capital letters A–Z.
o “Stewart is happy,” becomes: Hs
o This sort of statement is called a singular statement, because it refers to
an individual.

 When dealing with quantities of things (universal or particular) in


quantificational logic, individuals are replaced with lower case letters, x, y, and z.
 Universal claims are denoted with lower case letters, x, y, and z in
parentheses: (x), (y), or (z). These refer to “all x,” “all y,” or “all z.”
 Particular claims are called existential claims. These are denoted with a
backwards capital E, and lower case letters, x, y, and z, all of which are placed in
parentheses: (∃x), (∃y), or (∃z). These refer to “there exists an x,” “there exists
a y” or “there exists a z. &rdquo
 Putting your understanding of truth-functional notation together with your new
understanding of quantificational notation, you get the following symbolization:

o “All S are P” becomes, (x)(Sx ⊃ Px)


 “For all things x, if it’s S, then it’s P.”

o “No S are P” becomes, (x)(Sx ⊃ &##126; Px)


 “For all things x, if it’s S, then it’s not P.”

o “Some S are P” becomes, (∃x)(Sx • Px)


 “There is an x that’s both S and P.”

o “Some S are not P” becomes, (∃x)(Sx • ~ Px)


 “There is an x that’s an S but not a P.”

Four New Rules of Inference

 Universal instantiation is a predicate logic rule that allows you to remove the
universal quantifier, thereby allowing you to apply the implication rules and
replacement rules to derive the conclusion of a valid argument. UI says that,
since, for any x, if x is an S, then it’s a P, it follows that, if there’s an
unnamed x that’s an S, it’s also a P.
 Universal generalization is a predicate logic rule that allows you to add the
universal quantifier to an unquantified statement that does not refer to any
particular individual. UG says that, since any unnamed x that’s an S is also a P, it
follows that, for all x, if x is an S, then it’s a P.
 Existential instantiation is a predicate logic rule that allows you to remove the
existential quantifier, thereby allowing you to apply the implication rules and
replacement rules to derive the conclusion of a valid argument. EI says that,
since there is an x that’s both an S and a P, it follows that this particular S, who
we’ll call a is also a P.
o EI is a rule that allows you to pick an instance—an assumed
name˜without knowing just who or what that individual is. Suppose you

72
are outside a classroom door. You know there is a student in there, but
you don’t know who it is. So, you call that student, “a.” Doing so allows
you to move from the generalized claim, “There’s a student in the room,”
to picking out an unknown individual, in this case, “a.”
 Existential generalization is a predicate logic rule that allows you to add the
existential quantifier to an unquantified statement. EG says that, since there is
an unnamed x, or hypothetically named a that’s an S and a P, it follows that,
there exists an x that’s both an S and a P.

ASSESSMENT

Name:__________________________________________ Date:_______________
Year & Section:___________________________________ Rating:______________

Direction: Encircle the letter of the correct answer. Determine the correct translation of
the following statements.

1. Some dogs are not friendly. ( D, F )

a. ( x ) ( Dx • ~ Fx )
b. ~ ( ∃x ) ( Dx • ~ Fx )
c. ~ ( x ) ( Dx • ~ Fx )
d. ( ∃x ) ( Dx • ~ Fx )

2. Garfield is a fat cat. ( F,C )

a. ( g ) ( Fg • Cg )
b. FG • CG
c. Fg • Cg
d. Fg ⊃ Cg

3. Some cats are indoor pets. ( C,I )

a. ∃x ( Cx • Ix )
b. ( x ) ( Cx • Ix )
c. ( x ) ( Cx ⊃ Ix )
d. ∃x ( Cx ⊃ Ix )

4. Cats are good pets if they are spayed or neutered. ( C,P,S,N )

a. ( x ) { Cx ⊃ [ Px ⊃ ( Sx v Nx ) ] }
b. ( x ) { [ Cx • ( Sx v Nx ) ] ⊃ Px ) }

73
c. ∃x { Cx ⊃ [ Px ⊃ ( Sx • Nx ) ] }
d. ( x ) { Cx ⊃ [ Px ⊃ ( Sx • Nx ) ] }

5. Dogs bite if they are frightened or harassed. ( D,B,F,H )

a. ( x ) { [ Dx • ( Fx v Hx ) ] ⊃ Bx ) }
b. ( ∃x ) { Dx ⊃ [ Bx ⊃ ( Fx v Hx ) ] }
c. ( x ) { Dx v [ Bx ⊃ ( Fx v Hx ) ] }
d. ( ∃x ) ~ { Dx ⊃ [ Bx ⊃ ( Fx v Hx ) ] }
6. Dumbo is a strong vegan.
( Sx: x is strong; Vx: x is a vegan; d: Dumbo. )

a. Sd v Vd
b. ( x )Sd • Vd
c. Sd • Vd
d. Sd ⊃ Vd

7. All level twelve vegans are honest.


( Vx: x is a vegan; Lx: x is level twelve; Hx: x is honest. )

a. ( ∃x )[ ( Vx • Lx ) v Hx ]
b. ( x )[ ( Vx • Lx ) v ∃xHx ]
c. ( x )[ ( Vx • Lx ) ⊃ Hx ]
d. ( x )[ ( Vx ⊃ Lx ) ⊃ Hx ]

8. No vegan that isn’t an herbivore is honest.


( Vx: x is a vegan; Rx: x is an herbivore; Hx: x is honest. )

a. ( ∃ x )[( Vx • ~ Rx ) • Hx]
b. (∃ x ) [(~ Vx • ~ Rx ) • Hx]
c. ~ (∃ x ) [(~ Vx • ~ Rx ) • Hx]
d. ~ (∃ x ) [( Vx • ~ Rx ) • Hx]

9. Not all friends and family members of vegans are vegans.


( Vx: x is a vegan; Gxy: x is a friend of y; Fxy: x is a family member of y. )

a. ( x ){~ ( ∃y ) [ Vy • ( Gxy v Fxy ) ] ⊃ Vx }


b. ~ ( x ){ (∃y ) [ Vy ⊃ ( Gxy v Fxy ) ] ⊃ Vx }
c. ~ ( x ){ (∃y )[ ~ ( Vy • ( Gxy v Fxy ) ] ⊃ Vx }
d. ~ ( x ){ (∃y )[ ( Vy • ( Gxy v Fxy ) ] ⊃ Vx }

10. Not all friends of vegans are vegans.


( Vx: x is a vegan; Gxy: x is a friend of y )

74
a. ~ ( x ) { ( ∃y ) [ ( Vy • Gxy ) ≡ Vx ] }
b. ( x ) { (∃y ) [ ( Vy • Gxy ) ⊃ ~ Vx ] }
c. ~ ( x ){ (∃y ) [ ( Vy • Gxy ) ⊃ Vx ] }
d. ~ ( x ){ ( y ) [ ( Vy • Gxy ) ⊃ Vx ] }

LESSON X
ANALOGICAL REASONING
TOPICS
1. The Framework of Analogical Arguments
2. Analysing Analogical Arguments
3. Strategies of Evaluation

LEARNING OUTCOMES
At the end of the lesson, student should be able to:
 Expose students to different theories on the origin of knowledge and their
criticisms.

Analogical reasoning is one of the most fundamental tools used in creating an


argument. It can be analyzed as a type of inductive argument—it is a matter of
probability, based on experience, and it can be quite persuasive. Analogical reasoning
involves drawing an inference on the basis of similarities between two or more things.
To draw an analogy is simply to indicate that there are similarities between two or more
things.

An analogical argument is analyzed by revealing the general framework of the


argument. The argument lists the characteristics that two (or more) things have in
common and concludes that the things being compared probably have some other
characteristic in common. If an analogical argument is strong, then it raises the
probability that the conclusion is true.

There are three ways to show that an analogical argument is weak:

1. If you can point to something that is a direct result of an analogy, but that is
unacceptable to the person presenting the analogy, then you can put that
person in a difficult position. An unintended consequence of an analogy is
something that is a direct result of an analogy, but that is unacceptable to the
person presenting the analogy.

75
2. A disanalogy is another way in which an analogical inference can be weakened.
When we point out differences between two or more things, we create a
disanalogy.
3. When we create a counteranalogy, which is a new, competing argument
comparing the thing in question to something else, we cast some doubt on the
strength of the original analogical argument.

Four criteria are used to analyze the first premise of an analogical argument:

1. The strength of an analogical argument is related to the number of things


referred to in the first premise.
2. The strength of an analogical argument is related to the variety of things referred
to in the first premise.
3. The strength of an analogical argument is related to the number of
characteristics that are claimed to be similar between the things being
compared.
4. The strength of an analogical argument is related to the relevance of the
characteristics referred to in the first premise.

TOPIC 1: THE FRAMEWORK OF ANALOGICAL ARGUMENTS

An analogical argument relies on an analogy to draw a conclusion. An analogy is the


assertion of similarities (or dissimilarities) between two or more things. When we
reason analogically, we tend to use something well known to us in order to infer
something about a lesser known object or event. The fact that we compare similarities
between things tells us that we assume these things to be similar (or dissimilar) enough
to make it probable that these similarities (or dissimilarities) carry over:

The basic form of an analogical argument looks like this:

Premise 1: X and Y have characteristics a, b, c . . . in common.


Premise 2: X has characteristic k.
Therefore, probably Y has characteristic k.

TOPIC 2: ANALYZING ANALOGICAL ARGUMENTS

When we analyze an analogical argument, we look for relevant similarities and relevant
dissimilarities between the objects or events under comparison. The relevance of
similarities and dissimilarities between objects or events is determined by their relation
to the conclusion of the argument. For example, in the argument above about the
American Staffordshire terrier, it should be fairly clear that the dog’s markings bear no
relevance to whether the dog in the conclusion is going to be loyal.

The conclusion of an analogical argument is related to or determined by:

1) The number of things referred to in the first premise.


2) The variety of things referred to in the first premise.
3) The number of characteristics referred to in the first premise.
4) The relevance of the characteristics of things referred to in the first premise.

76
TOPIC 3: STRATEGIES OF EVALUATION

When evaluating an analogical argument, consider:

1) Disanalogies. These are differences between things, or ways in which things are not
similar.
2) Counteranalogy. This is a new, competing argument which compares the conclusion
object or event to something else.
3) Unintended consequences. When you can point to an undesirable consequence of
the analogy someone has advanced, they will be less likely to maintain it.

Explore

The Framework of Analogical Arguments

 Analogical arguments typically help us to draw an inference about some object


or event that’s not very well known, from an object or event that is. The strength
or weakness of the argument depends in large part on how many relevant
features the two objects or events share in common.

Analyzing Analogical Arguments

 People often criticize an analogical argument by saying, “That’s like comparing


apples and oranges.” What they mean is that two things shouldn’t be compared
to each other if they aren’t alike. However, depending on the attributes one
thing (A) shares with another (B), we may be warranted in using A as a model for
B.
 To appreciate the importance of analogical arguments to other types of
reasoning, consider the relationship analogical reasoning has with inductive
generalization. In order to infer something about an entire class of things based
on a portion of that class, you must have already concluded that the things in the
class share enough relevant features to be included as members of that class.
This means that analogical arguments precede generalizations.

Strategies of Evaluation

 When reasoning analogically, make a list (something like a “pro/con” format) of


the relevant attributes the two things have in common under one column, and a
list of relevant attributes the two things don’t have in common under another
column. For example, suppose you’re trying to decide if you should get a third
dog. Your friend already has three dogs. There are quite a few variables to
consider. Here’s how your list might look:

77
Notice that the two situations are rather different. You have to ask yourself if there are
enough relevant similarities between your situation and your friend’s situation to make
a good comparison. You also have to ask yourself if there are relevant dissimilarities.

ASSESSMENT

Name:__________________________________________ Date:_______________
Year & Section:___________________________________ Rating:______________

Direction: Encircle the letter of the correct answer.

Determine the correct framework for the following analogical arguments.

1. The new Batman movie is exceeding all expectations of box office earnings. The
last box office smash won the Oscar for best picture. So, the new Batman movie
is going to win the Oscar for Best Picture.

a. The new Batman movie (Y) has made a lot of money, as did the
last box office smash (X). X won the Oscar for Best Picture, so Y will also
win the Oscar for Best Picture.
b. The new Batman movie (X) has made a lot of money, as did the
last box office smash (Y). X won the Oscar for Best Picture, so Y will also
win the Oscar for Best Picture.
c. The new Batman movie (X) has made a lot of money, as did the
last box office smash (Y). Y won the Oscar for Best Picture.
d. The new Batman movie (X) has made a lot of money, as did the
last box office smash (Y). So X will win the Oscar for Best Picture.

78
2. I don’t understand why the military can’t get along without more money. We’re
all tightening our belts, and the military is no different from any other
organization on a budget. The military has to purchase and maintain equipment,
just like my manufacturing plant has to do. The military has to make payroll, just
like my manufacturing plant has to do. Since my manufacturing plant just had to
reduce its budget by 20%, the military should reduce its budget by 20%, too.

a. The military (Y) has to purchase and maintain equipment, just like
my manufacturing plant has to do (X); both X and Y have to make payroll;
Y just had to reduce its budget by 20%, so (X) should reduce its budget by
20%, too.
b. The military (X) has to purchase and maintain equipment, just like
my manufacturing plant has to do (Y); both X and Y have to make payroll;
Y just had to reduce its budget by 20%.
c. The military (X) has to purchase and maintain equipment, just like
my manufacturing plant has to do (Y); X just had to reduce its budget by
20%, so (Y) should reduce its budget by 20%, too.
d. The military (Y) has to purchase and maintain equipment, just like
my manufacturing plant has to do (X); both X and Y have to make payroll;
X just had to reduce its budget by 20%, so (Y) should reduce its budget by
20%, too.

The following passage has been analyzed according to the four criteria in Section 10.B.
All are correct except for one. Identify the incorrect analysis.

3. If you like Russ’s Steakhouse, then you’ll like Mutt’s Burgers! Mutt gets his burger
meat from the same supplier that Russ gets his steaks. Russ gets his meat fresh daily.
So does Mutt. Russ grills his steaks. Mutt grills his burgers. Come to Mutt’s today!

a. Number of entities: Steak meat and burger meat


b. Variety of instances: Steaks
c. Number of characteristics: Same supplier, fresh meat daily, grilled
meat
d. Relevancy: Apart from the type of beef, the characteristics are
relevant

4. I am sure I will like this piece of fish. After all, I love sea bass, which has no bones.
This piece of fish has no bones, either. In addition, both this piece of fish and sea
bass have delicate white flesh. This piece of fish and sea bass also taste a lot like
lobster.

a. Number of entities: One piece of fish and the class of sea bass
b. Variety of instances: All are fish.
c. Number of characteristics: No bones, delicate white flesh, taste
like lobster
d. Relevancy: Tasting like lobster is not relevant to the comparison.

79
The following analogical arguments have been evaluated using the three strategies
discussed in Section 10C. Determine whether the additional information strengthens or
weakens the argument, or is not relevant to it.

5. The new Batman movie is exceeding all expectations of box office earnings. The
last box office smash won the Oscar for best picture. So, the new Batman movie is
going to win the Oscar for Best Picture.

Additional information: The last box office smash was based on a comic book
hero.

a. Strengthens
b. Weakens
c. Not relevant

LESSON XI
LEGAL ARGUMENTS

TOPICS
1. Legal arguments and its concepts.

LEARNING OUTCOMES
At the end of the lesson, student should be able to:
 Allow students to critique and provide precise and perspicuous argument.

TOPIC 1: LEGAL ARGUMENTS AND ITS CONCEPTS

As with all arguments, legal arguments contain at least one premise and can be
appreciated and understood when you are able to grasp the underlying logic. Legal
discourse has evolved patterns and conventions that we can recognize and apply to
specific legal cases. Legal reasoning is also called “rule-based reasoning,” which means
that it follows specific rules. For example, a rule that specifies a test with mandatory
elements lists all the necessary conditions that must be met in order for the rule to be
applicable.

Legal reasoning proceeds either deductively or inductively, but it is important to


remember that these concepts are defined in legal reasoning rather more narrowly than
they are in logic. In law, deductive reasoning generally means going from the general to

80
the specific—that is, from the statement of a rule to its application to a particular legal
case. Many law textbooks refer to inductive reasoning as the process of going from
the specific to the general. It comes into play whenever we move from a specific case or
legal opinion to a general rule. One type of experiential reasoning studied in Chapter 10
(analogical reasoning), is crucial to legal reasoning, but does not fit the standard legal
definition of inductive reasoning. Nevertheless, legal reasoning, relying as it does on
precedent (a judicial decision that can be applied to later cases) and similar cases, often
relies on analogies.

There are several important concepts in legal reasoning with which you should be
conversant. They include the following:

 Appellate courts: Courts of appeal which review the decisions of lower courts.
 Probative value: Evidence that can be used during a trial to advance the facts of
the case.
 Prejudicial effect: Evidence that might cause some jurors to be negatively biased
toward a defendant.
 Indictment: A formal accusation presented by a grand jury.
 Affidavit: A written statement signed before an authorized official.
 Rules of law: The legal principles that have been applied to historical cases.
 Plaintiff: The person who initiates a lawsuit.

Understanding the basic features of truth functional logic will aid you in analyzing legal
arguments. The truth functional connectives, conditional, conjunction, and disjunction,
along with the logical notation for simple sentences, serve to reveal the form of a piece
of legal reasoning.

A. Deductive and Inductive Reasoning. The concepts of deductive and inductive


reasoning in legal arguments are narrower than they are in other contexts. Deductive
arguments in legal reasoning are said to proceed from the general to the specific, i.e.,
from rule to application to a specific case. Inductive arguments in legal reasoning are
said to proceed from the specific to the general, i.e., from a specific case or a legal
opinion to a rule. Whichever way the argument proceeds, however, legal reasoning is
called rule-based reasoning.

B. Conditional Statements. Conditional statements typically appear as “if . . . then . . . ”


statements in legal reasoning. In addition, phrases and words such as “every time,”
“whenever,” “all cases where,” “given that,” and “in the event of” also reflect
conditional structure.

C. Necessary and Sufficient Conditions. We saw in Chapter 3 that there are two types of
condition associated with conditional statements: necessary and sufficient. Typically,
the order is as follows: “If sufficient condition, then necessary condition.” Elements of
laws are often constructed in this way.

D. Disjunction and Conjunction. Disjunctions are used as tests in legal reasoning for
establishing a sufficient condition: Satisfying at least one of the disjuncts is sufficient for
something else to obtain. Posting bail or the charges being dropped is sufficient for a
prisoner to be released from custody.

Conjunctions are used as tests in legal reasoning for establishing a necessary condition:
Both conjuncts must be satisfied for something else to obtain. One must be charged and
convicted in order to serve a lawful prison sentence.

81
E. Analogies. Analogies are used in legal reasoning in two ways: precedent and rules of
law. A precedent is a judicial decision that can be applied to subsequent cases, while
rules of law are the legal principles that have been applied to historical cases. Lawyers
are typically interested in finding precedents that are both similar enough to and
support that lawyer’s position in the current case. Through analogical reasoning, they
must show that the facts of their case are similar to the precedent case to which they
appeal. They are also interested in finding rules of law that are similar enough to the
current case.

When you analyze a legal argument, you should look for analogical reasoning according
to the structures and strategies you learned in Chapter 11, along with the specific type
of analogy that appears—precedent or rules of law.

Explore

Conditional Statements

 Identifying the ways a conditional statement is expressed will help you


understand legal reasoning. “If-then,” “if,” “only if,” “whatever,” “whenever,”
every time,” “given that,” “all cases where,” and “in the event that,” are
examples of how a conditional statement can be expressed.

Necessary and Sufficient Conditions

 Understanding how the parts of a conditional statement work is important to


understanding the function of conditional statements in legal reasoning. The
antecedent of a conditional statement asserts a sufficient condition, while the
consequent asserts a necessary condition.
 Recall from Chapter 3 that a sufficient condition is one that guarantees a
happening. In a conditional statement, a sufficient condition is such that a true
antecedent guarantees a true consequent. When a law asserts, “If X, then Y,”
whatever X is, is claimed a sufficient condition for Y.
 Recall from Chapter 3 that a necessary condition is one that must obtain for
something else to occur. Absent that condition, the something else will not
occur. Being a citizen of the United States is a necessary condition for voting in
an election.

Analyzing a Complex Rule

 Enlist your knowledge of analyzing truth functional ordinary language


statements. You should let capital letters stand for simple statements, and group
compound elements together by way of parentheses. Identify significant
connectives: conditional, disjunction, and conjunction. Looking at the form of
legal reasoning will aid you in working past the often dense sentence
constructions.

Analogies

 You already know from Chapter 10 that analogical reasoning is a powerful tool
for making progress in understanding complicated arguments. Legal reasoning
uses analogies by way of comparisons between an existing case and a rule of
law. As with analyses of other analogical arguments, look for the relevant
similarities and dissimilarities between the case at hand and the law.

82
ASSESSMENT

Name:__________________________________________ Date:_______________
Year & Section:___________________________________ Rating:______________

Direction: Encircle the letter of the correct answer.

Analyze the logical apparatus used in the particular rules of evidence below. Follow the
method of analysis that we did for Rule 609(a) of the Federal Rules of Evidence.
Highlight any logical operators when available. Rewrite and reconstruct the statements
whenever necessary in order to reveal the logic of the rule. Then answer the question
that follows.

1. Rule 413. Evidence of Similar Crimes in Sexual Assault Cases


In a criminal case in which the defendant is accused of an offense of sexual
assault, evidence of the defendant's commission of another offense or offenses
of sexual assault is admissible, and may be considered for its bearing on any
matter to which it is relevant.
Assuming the following key:

(A) In a criminal case in which the defendant is accused of an offense of sexual


assault, evidence of the defendant's commission of another offense or offenses
of sexual assault is admissible.
(B) In a criminal case in which the defendant is accused of an offense of sexual
assault, evidence of the defendant's commission of another offense or offenses
of sexual assault may be considered for its bearing on any matter to which it is
relevant.

Which of the following best captures the logical structure of the argument?

83
a. A and B
b. A or B
c. If A then B
d. If B then A

2. Rule 104: Preliminary Questions


Relevancy conditioned on fact.—When the relevancy of evidence depends upon
the fulfillment of a condition of fact, the court shall admit it upon, or subject to,
the introduction of evidence sufficient to support a finding of the fulfillment of
the condition.

Assuming the following key:

(A) The relevancy of evidence depends upon the fulfillment of a condition of fact.
(B) The court shall admit it.
(C) Evidence sufficient to support a finding of the fulfillment of the condition is
introduced.

Which of the following best captures the logical structure of the argument?

a. If A, then, (if B then C)


b. If C, then, (if B then A)
c. If (C, then, if B) then A)
d. If B, then, (if A then C)

Analyze the logical apparatus used in the particular rules of evidence below. Follow the
method of analysis that we did for Rule 609(a) of the Federal Rules of Evidence.
Highlight any logical operators when available. Rewrite and reconstruct the statements
whenever necessary in order to reveal the logic of the rule. Then answer the question
that follows.
3. Rule 413. Evidence of Similar Crimes in Sexual Assault Cases
In a criminal case in which the defendant is accused of an offense of sexual
assault, evidence of the defendant's commission of another offense or offenses
of sexual assault is admissible, and may be considered for its bearing on any
matter to which it is relevant.
Assuming the following key:

(A) In a criminal case in which the defendant is accused of an offense of sexual


assault, evidence of the defendant's commission of another offense or offenses
of sexual assault is admissible.
(B) In a criminal case in which the defendant is accused of an offense of sexual
assault, evidence of the defendant's commission of another offense or offenses
of sexual assault may be considered for its bearing on any matter to which it is
relevant.

Which of the following best captures the logical structure of the argument?

84
a. A and B
b. A or B
c. If A then B
d. If B then A

LESSON XII
MORAL ARGUMENTS

TOPIC
1. Value Judgments
2. Moral Theories
3. The Naturalistic Fallacy
4. Structure Moral Arguments
5. Analogies and Moral Arguments

LEARNING OUTCOMES
At the end of the lesson, student should be able to:
 To involve students in the ideas of rights and obligations.

Some arguments rely solely on factual claims for support, some arguments rely solely on
value judgments for support, and some arguments rely on a mixture of the two. A value
judgment is a claim that a particular human action or object has some degree of
importance, worth, or desirability. We engage in moral reasoning when we construct or
follow arguments that rely on value judgments exclusively, or a mixture or value
judgments and descriptive judgments.

Some moral arguments are intended to be prescriptive. In a moral setting, advice may
be offered either by specifying a particular action that ought to be performed, or by
providing general moral rules, principles, or guidelines that should be followed. Some

85
moral arguments are normative. Normative arguments establish standards for correct
moral behavior; determining norms or rules of conduct.

When studying moral arguments, it is important to be conversant with the following


concepts:

 Emotivism: Holds that moral value judgments are merely expressions of our
attitudes or emotions.
 Consequentialism: A class of moral theories in which the moral value of any
human action or behavior is determined exclusively by its outcomes.
 Teleology: The philosophical belief that the value of an action or object can be
determined by looking at the purpose or the end of the action or object.
 Egoism: The basic principle that everyone should act in order to maximize his or
her own individual pleasure or happiness.
 The most important principle for utilitarianism can be summed up in the famous
dictum “the greatest good for the greatest number.””
 According to utilitarianism, every human action is “universalizable,” meaning
that the same principles hold for all people at all times.
 Deontology: Holds that duty to others is the first and foremost moral
consideration, and it lays the groundwork for discovering those duties.
 The basic idea of a categorical imperative is that your actions or behavior toward
others should always be such that you would want everyone to act in the same
manner.
 Situation ethics agrees that there can be general, even objective, moral rules.
However, it holds that we should not rigidly apply those rules to every possible
situation.
 Relativism makes two claims: First, all moral value judgments are determined by
an individual’s personal beliefs or by a society’s beliefs toward actions or
behavior. Second, there are no objective or universal moral value judgments.
 Naturalistic moral principle: Since it is natural for humans to desire pleasure (or
happiness) and to avoid pain, human behavior ought to be directed to these two
ends.
 Naturalistic fallacy: Value judgments cannot be logically derived from statements
of fact.

TOPIC 1: VALUE JUDGMENTS

Moral arguments enlist both descriptive and value judgments. Descriptive judgments
are statements that describe. Value judgments are statements that reflect a belief in the
value, worth, importance, or desirability of an object or a human action. Very often,
value judgments are prescriptive, in that they assert that one should do or believe
something. Just as often, value judgments are normative, in that they assert a standard
for correct moral behavior or rules of conduct.

Value Judgments

 Bear in mind the distinction between descriptive and value judgments. A


descriptive judgment is an attempt to state the facts, or what is the case. A value
judgment involves an evaluation of the quality of the facts, or what is the case.
Value judgments typically involve adjectives like “good” and “bad.”

86
TOPIC 2: MORAL THEORIES

There are myriad moral theories, but major theories can be classified as follows:

1. Emotivism: The theory that moral value judgments are merely expressions of
one’s attitudes or emotions.
2. Consequentialism: The theory that moral value is determined by the outcomes
or consequences of actions.
a. Teleology is a type of consequentialism that asserts value can be
determined by ascertaining the purpose of an object or action.
b. Egoism is a type of consequentialism which asserts that everyone should
act in order to increase his or her own pleasure or happiness.
c. Utilitarianism is a type of consequentialism which asserts that moral
value is determined by how much happiness is produced for the greatest
number of people.

Moral Theories

 One way to think about a moral theory is in terms of its classification as


consequentialist or non-consequentialist. Consequentialist moral theories root
the moral value of an action or belief in the quality of the consequences
generated. An action that generates good consequences means the action was
good. An action that generates bad consequences means the action was bad.
Non-consequentialist moral theories root the value of an action or belief in
something other than consequences. The concept of duty is an example of a
value that is not related to consequences.

3. Deontology: A duty-based ethical theory that morality is determined by one’s


duty to others.
a. The first formulation of Kant’s categorical imperative is the exemplar of
deontological expressions: “Act only on that maxim that you can will at
the same time to become a universal law.”
4. Situation Ethics: The theory that moral value may be generalized but not
absolute.
5. Relativism: A two-part theory that moral value is determined by the individual’s
personal beliefs or society’s beliefs about conduct; there are no objective moral
values.

TOPIC 3: THE NATURALISTIC FALLACY

Although it may be true that human beings naturally desire their own pleasure or
happiness and naturally avoid pain, it does not follow that pleasure or pain ought to be
pursued or avoided. Some moral theorists infer from the fact we do pursue pleasure
and avoid pain that we ought to do so. (This is called the naturalistic moral principle.)
Other moral theorists, the ones who argue that the inference is erroneous, accuse the
naturalistic moral philosophers of committing the naturalistic fallacy. We bear this
debate in mind as we analyze certain sorts of moral arguments.

TOPIC 4: THE STRUCTURE OF MORAL ARGUMENTS

87
We can detect a moral argument when we have an argument that contains:
1) “At least one premise describing a particular situation where a decision to act will be
made by someone”;
2) “at least one premise that supplies a moral rule, principle, or command”;
3) and “the conclusion asserts that a specific action should be performed” .

TOPIC 5: ANALOGIES OF MORAL ARGUMENTS

Analogical reasoning appears in every type of reasoning, including moral arguments. It


functions the same way in moral arguments as it does elsewhere, except the conclusion
involves a moral judgment. Typically, the comparison involves a morally neutral object
or event and the moral object or event of the conclusion. Lastly, analogical reasoning is
often the vehicle for moral arguments because they do a good job of pointing out
inconsistent moral views or behaviors.

ASSESSMENT

Name:__________________________________________ Date:_______________
Year & Section:___________________________________ Rating:______________

Direction: Encircle the letter of the correct answer.

Determine whether the following statement is a factual claim or a value claim. If a


statement makes a value claim, determine if it is a moral value claim or a personal value
claim.
1. One ought not use another’s prescription medication.
a. Factual claim
b. Moral value claim
c. Personal value claim
2. If more yellow cab licenses are given out, traffic will become more congested in
New York City.
a. Factual claim
b. Moral value claim
c. Personal value claim
3. The Olympics is just another devious way for big corporations to make billions of
dollars.
a. Factual claim
b. Moral value claim
c. Personal value claim

88
Read the following argument and answer the question that follows.
4. We should support the state lottery because it helps education. A portion of the
money you spend on the lottery goes to pay for books, supplies, and other things
schools need. The better our schools are, the better society will be.

Which of the following best describes the argument?


a. Relativist
b. Deontological
c. Consequentialist
d. Emotivist

5. One ought to give to charities when one can, according to one’s means, and
without expecting anything in return. This is to say, one ought to give when one is
able, give no more than one is able, and not anticipate reward. Suppose you are
driving down a lonely road and see a child. No one is with that child, and the child
looks hurt. You don’t have to stop and help—you could keep driving—but you
believe you must. This is an example of giving. One ought to give because it is the
right thing to do.

Which of the following best describes the argument?


a. Teleological
b. Deontological
c. Consequentialist
d. Emotivist

Determine whether the following statement is a factual claim or a value claim. If a


statement makes a value claim, determine if it is a moral value claim or a personal value
claim.
7. If more yellow cab licenses are given out, traffic will become more congested in
New York City.
a. Factual claim
b. Moral value claim
c. Personal value claim
8. The Olympics is just another devious way for big corporations to make billions of
dollars.
a. Factual claim
b. Moral value claim
c. Personal value claim

Read the following argument and answer the question that follows.
9. We should support the state lottery because it helps education. A portion of the
money you spend on the lottery goes to pay for books, supplies, and other things
schools need. The better our schools are, the better society will be.

Which of the following best describes the argument?

89
a. Relativist
b. Deontological
c. Consequentialist
d. Emotivist

10. One ought to give to charities when one can, according to one’s means, and
without expecting anything in return. This is to say, one ought to give when one is
able, give no more than one is able, and not anticipate reward. Suppose you are
driving down a lonely road and see a child. No one is with that child, and the child
looks hurt. You don’t have to stop and help—you could keep driving—but you
believe you must. This is an example of giving. One ought to give because it is the
right thing to do.

Which of the following best describes the argument?


a. Teleological
b. Deontological
c. Consequentialist
d. Emotivist

LESSON XIII
STATISTICAL ARGUMENTS AND PROBABILITY

TOPIC
1. Samples and Populations
2. Statistical Averages
3. Standard Deviation
4. The Misuse of Statistics

LEARNING OUTCOMES
At the end of the lesson, student should be able to:
 To identify arguments subject to study of probability and statistical
inference.

Statistics is the methodological study of numeric data. We use statistical results in our
daily lives, and statistics are often used as evidence in arguments. Evaluating arguments
that rely on statistical evidence requires that we can correctly interpret the statistical
evidence as it is presented. For this reason, it is important to understand how statistics
works.

At the heart of statistical reasoning is the concept of probability, that is, the likelihood
that some event happens or is the case. Statistics is the study of numerical probability.
Good statistical inferences rely on deriving legitimate numerical data. Determining the

90
likelihood that a candidate will be elected to office involves understanding what, for
example, “sample size” means.

There are multiple approaches to generating statistical data used to make inferences:

 The restricted conjunction method is used in situations dealing with two or more
independent events, where the occurrence of one event has no bearing
whatsoever on the occurrence or nonoccurrence of the other event.
 The general conjunction method is used for calculating the probability of two or
more events occurring together, regardless of whether the events are
independent or not independent.
 The restricted disjunction method is used when two (or more) events are
independent of each other, and the events are mutually exclusive (if one event
occurs then the other cannot).
 The general disjunction method is used for calculating the probability of
occurrence of two or more events that are not mutually exclusive.
 The negation method is used once the probability of an event occurring is
known; it is then easy to calculate the probability of the event not occurring.
 Probability calculations using the a priori theory rely on hypothetical reasoning
based on two major assumptions. The first is that all the possible outcomes can
be determined, and the second is that each of the possible outcomes has an
equal probability of occurring (equiprobable).
 In the relative frequency theory, some probabilities can be computed by dividing
the number of favorable cases by the total number of observed cases.
 Reliance on statistical, historical data is not the same as relative frequency
applications. Calculations of this kind fall under the subjective theory of
probability, where determinations are based on the lack of total knowledge
regarding an event.

Fluency with the following terms and definitions is important to a working knowledge of
statistics:

 Population: Any group of objects, not just human populations.


 Sample: A subset of a population.
 Representative sample: A sample that accurately reflects the characteristics of
the population as a whole.
 Random sample: When every member of the population has an equal chance of
getting into the sample.
 Mean: The statistical average that is determined by adding the numerical values
in the data concerning the examined objects, then dividing by the number of
objects that were measured.
 Median: The statistical average that is determined by locating the value that
separates the entire set of data in half.
 Mode: The statistical average that is determined by locating the value that
occurs most.
 Standard deviation: Describes the amount of diversity in a set of numerical
values.
 Probability calculus: A branch of mathematics that can be used to compute the
probabilities of complex events from the probabilities of their component
events.
 Conditional probability: The calculation of the probability of an event if another
event has already happened.

91
Statistical reasoning allows us to use specific inductive principles and mathematical
calculations to generate conclusions about events and groups of things. Although
mathematics is a deductive science, it can be used to generate inductive conclusions.
This is because of the types of generalizations inferred from the calculations, and
because of the fact that the guiding principles are fundamentally experiential.

TOPIC 1: SAMPLES AND POPULATIONS

When we want to know something about a specific group of things, we use samples (a
portion of a population) to generalize from specific instances to the target. We do this
because we cannot successfully count each individual member of the target group. For
example, if we want to know something about “U.S. citizens’ attitudes toward
immigration,” we can’t interview each one. What we do, instead, is identify
our population, which in this case is U.S. citizenry and determine the representative
sample, that is, the “sample that accurately reflects the characteristics of the population
as a whole”. Sometimes, the representative sample is created through a random
sample, in which “every member of the population has an equal chance of getting into
the sample”.

TOPIC 2: STATISTICAL AVERAGES

Very often, our aim is to find a statistical average—a numeric version of generalization.
There are three types of statistical average:

1) Mean: determined by adding relevant numerical values, then dividing by the number
of objects represented by those values.
2) Median: determined by locating the number that divides the numeric data set in half.
3) Mode: determined by locating the value in the data set that occurs most.

TOPIC 3: STANDARD DEVIATION

Standard deviation measures the amount of diversity in a numeric data set. This is
useful in statistical reasoning for determining diversity around the mean.

The standard deviation of a numeric data set is calculated through a series of steps.

TOPIC 4: THE MISUSE OF STATISTICS

As with any reasoning, statistical inferences can be erroneous. This can be accidental or
intentional, but in either case, it is important never to take conclusions inferred from
statistical evidence at face value. (See the fallacy of misleading precision in Chapter 4.)

 Understanding how evidence is gathered, statistical figures computed, and


results interpreted is essential to following and evaluating statistical and
probabilistic arguments. You should be conversant with important concepts such
as population, sample, representative sample, random sample, and standard
deviation, as well as the various ways in which statistical averages are calculated.
Understanding the concepts will make it much easier for you to learn the various
formulas enlisted in statistical and probabilistic reasoning.

92
ASSESSMENT

Name:__________________________________________ Date:_______________
Year & Section:___________________________________ Rating:______________

Direction: Encircle the letter of the correct answer.


1. The Olympics is just another devious way for big corporations to make billions of
dollars.
a. Factual claim
b. Moral value claim
c. Personal value claim

2. Read the passage that follows. Identify the sample size, and whether or not the
sample is representative of the population referred to in the conclusion.

Nine out of 10 students surveyed claimed the graduation requirements for a


baccalaureate degree at Quindle College were too stringent. Therefore,
graduation requirements at colleges nationwide are too stringent. Ninety
percent of students can’t be wrong, so college administrators and academic
departments need to revisit their degree programs.
a. Sample size: We are not given the sample size. “Nine out of 10”
could mean 10 people, 100 people, 1,000 people, etc., took the survey.
b. Sample size: The sample size is 10. Since college student are very
much alike, it is likely the sample is representative of the population.
c. Sample size: We are not given the sample size. “Nine out of 10”

93
could mean 10 people, 100 people, 1,000 people, etc., took the survey. If
the sample is very small, the generalization to all college students is
erroneous, i.e., it’s unlikely the sample is representative of the
population.
d. Sample size: The sample size is 10. If the sample is very small, the
generalization to all college students is erroneous, i.e., it’s unlikely the
sample is representative of the population.

3. Read the passage that follows. Identify the correct sample and population.

Nine out of 10 students surveyed claimed the graduation requirements for a


baccalaureate degree at Quindle College were too stringent. Therefore,
graduation requirements at colleges nationwide are too stringent. Ninety
percent of students can’t be wrong, so college administrators and academic
departments need to revisit their degree programs.
a. Sample: College administrators and academic departments
Population: College students nationwide
b. Sample: College students nationwide
Population: College administrators and academic departments
c. Sample: College students nationwide
Population: Students surveyed at Quindle College
d. Sample: Students surveyed at Quindle College
Population: College students nationwide

4. Read the passage that follows. Identify the sample size, and whether or not the
sample is representative of the population referred to in the conclusion.

In the last year, more than 5000 people in the northeastern United States have
contracted a rare, severe, and sometimes fatal tick disease called babesiosis.
The microscopic organism infects red blood cells, and symptoms take anywhere
from one to eight weeks to manifest. Researchers have found that 62% of the
5000 people infected with babesiosis had pre-existing autoimmune disorders,
while the number of otherwise healthy individuals was insignificant. Thus, we
can conclude that people with autoimmune problems are especially vulnerable
to this tick-borne disease.
a. Sample size: The size is large enough, and we are told the source
of the data, namely people in the U.S. infected with babesiosis. The
population in the conclusion is the class of people with autoimmune
problems, but since the sample is taken from an area where the
concentration of tick-borne diseases is high, it’s not clear that the sample
is representative.
b. Sample size: The size is large enough, but we are not told the
source of the data. Since the sample size is large enough, the sample is
representative. The population in the conclusion is the class of people
with autoimmune problems, but since the sample is taken from an area
where the concentration of tick-borne diseases is high, it’s not clear that
the sample is representative.
c. Sample size: The size is not large enough, and we are not told the
source of the data. Since the sample size is not large enough, the sample
is not representative.

94
d. Sample size: The size is large enough. The population in the
conclusion is the U.S. population, but since the sample consists of people
with autoimmune disease, the sample is not representative.

5. Read the passage that follows. Identify the correct sample and population.
In the last year, more than 5000 people in the northeastern United States have
contracted a rare, severe, and sometimes fatal tick disease called babesiosis.
The microscopic organism infects red blood cells, and symptoms take anywhere
from one to eight weeks to manifest. Researchers have found that 62% of the
5000 people infected with babesiosis had pre-existing autoimmune disorders,
while the number of otherwise healthy individuals was insignificant. Thus, we
can conclude that people with autoimmune problems are especially vulnerable
to this tick-borne disease.
a. Sample: All people
Population: 5000 people
b. Sample: 5000 people
Population: All people
c. Sample: 62% of the population
Population: All people
d. Sample: People with autoimmune disease
Population: All people

LESSON XIV
CAUSALITY AND SCIENTIFIC ARGUMENTS
TOPIC
1. Causality
2. Mill’s Methods
3. Theoretical and Experimental Science
4. Inference to the Best Explanation
5. Hypothesis Testing, Experiments, and Predictions
6. Science and Superstition

LEARNING OUTCOMES
At the end of the lesson, student should be able to:
 To apply critical thinking skills to perform reasoning and problem solving.

The word “cause” has several meanings, and in everyday situations the possibility of
ambiguity arises. Theories of causation are, therefore, beneficial to study, for although
they do not yield a definitive account of the concept, they clarify our thinking by
providing conceptual frameworks within which we can think about causality and cause
and effect relationships. So, although Mill’s methods do not provide conclusive proof of
causality, the five methods are helpful in discovering correlations and potential causes.

95
The scientific method is built on the concept of causation. Theoretical science proposes
explanations—hypotheses—for natural phenomena, while experimental science tests
those explanations. A good hypothesis provides an explanation for known facts and a
way to test an explanation.

We can test a hypothesis by getting it to make a prediction. Predictions are either true
or false; the results are used to confirm (support) or disconfirm (refute) the hypothesis.

Three requirements ensure a fair test of the causal hypothesis:

1. The prediction should be something that is verifiable.


2. The prediction should be unlikely to be true based on our background
knowledge.
3. There should be a connection between the hypothesis and the prediction.

Five criteria need to be considered to fully appreciate the complexity of causality:

1. There should be a correlation between the cause and the effect.


2. The cause should precede the effect.
3. The cause should be in the proximity of the effect.
4. A set of necessary and sufficient conditions should exist.
5. Alternative explanations should be ruled out.

The weight of our answers to all five criteria together establishes the grounds for a
satisfactory cause-effect relationship. The weight we give to a confirmation or refutation
is never all-or-nothing. We need to accumulate evidence over a long time.

Being conversant with following concepts is important for a successful understanding of


causality and scientific method:

 Causal network: A set of conditions that bring about an effect.


 Normal state: The historical information regarding an object.
 Abnormal state: A drastic change in the normal state. Any change from the
normal state requires an explanation, typically a causal one.
 Precipitating cause: The object or event directly involved in bringing about an
effect.
 Remote cause: Something that is connected to the precipitating cause by a chain
of events.
 Mill’s method of agreement: The method that looks at two or more instances of
an event to see what they have in common.
 Mill’s method of difference: The method that looks for what all the instances of
an event do not have in common.
 Mill’s joint method of agreement and difference: If two or more instances of an
event have only one thing in common, while the instances in which it
does not occur all share the absence of that thing, then the item is a likely cause.
 Mill’s method of residues: The method that subtracts from a complex set of
events those parts that already have known causes.
 Mill’s method of concomitant variations: The method that looks for two factors
that vary together.
 Correlation: A correspondence between two sets of objects, events, or sets of
data.
 The process of abduction occurs when we infer explanations for certain facts.

96
 Inference to the best explanation: When we reason from the premise that a
hypothesis would explain certain facts to the conclusion that the hypothesis is
the best explanation for those facts.
 Controlled experiment: One in which multiple experimental setups differ by only
one variable.
 Experimental group: The group that gets the variable being tested.
 Control group: The group in which the variable being tested is withheld.
 Verifiable prediction: The prediction, if it is true, must include an observable
event.
 Nontrivial prediction: Requires reference to background knowledge, which is
everything we know to be true.

Scientific reasoning proceeds on the assumption that there are discernable causal
relations between objects and events. What causality is, however, is not as clear as you
might think.

TOPIC 1: CAUSALITY

A cause can be defined as a condition or set of conditions that bring about an effect.
When we talk about a set of conditions, we are talking about a causal network.

At the heart of all scientific endeavors, including scientific reasoning, is the concept of
causality. To connect two or more events by this concept is to claim an advance in
knowledge. But what is causality? There are multiple theories of causality, some of the
most important of which you are studying in Chapter 14. It is important that you bear
this fact in mind as you proceed, as thinking about each theory as a way of
understanding the concept of causality will help you better understand how scientific
reasoning works.

TOPIC 2: MILL’S METHODS

Accurately determining causes and effects is not a simple task. We can often confuse
the two or misidentify one because we lack sufficient information. Mill’s methods are
attempts to isolate a cause from a complex event sequence.

1) Method of agreement: Two or more instances of an event (effect) are compared to


see what they have in common. That commonality is identified as the cause.
2) Method of difference: Two or more instances of an event (effect) are compared to
see what they all do not have in common. If they have all but one thing in common, that
one thing is identified as the cause.
3) Joint method of agreement and difference: A combination of the methods of
agreement and difference, the joint method looks for a single commonality among two
or more instances of an event, and the joint method looks for a common absence of
that possible cause.
4) Method of residues: All known causes of a complex set of events are subtracted.
What is leftover is said to be the cause.
5) Method of concomitant variations: Correlations between varying events are sought,
i.e., correspondence in variations between two sets of objects, events, or data.

Mill’s five methods of determining causality all assume that events can be understood
by establishing a specific relationship between them. A further assumption is that this

97
new understanding will bear on future, similar events. So, each of the five methods
offers us a way of sifting through the complexities of events to isolate the condition or
conditions that brought about a current state of affairs.

TOPIC 3: THEORETICAL AND EXPERIMENTAL SCIENCE

Scientists proceed by developing a hypothesis from observed data. A hypothesis is a


provisional and testable explanation for facts. Theoretical scientists propose
hypotheses for natural phenomena, while experimental scientists conduct tests of
those hypotheses.

A hypothesis is an explanation of a known fact or set of facts, and it is also a way of


testing that explanation for use in further discoveries. You might also consider it this
way: a hypothesis is a way of thinking about a cause.

TOPIC 4: INFERENCE TO THE BEST EXPLANATION

The process whereby a hypothesis is developed is called abduction. Inference from facts
to an explanation of those facts, particularly where patterns occur, is an abductive
inference. In order to resolve conflicting inferences for the same facts, we often have
recourse to inference to the best explanation, which is to say, when the inference is the
most probable.

TOPIC 5: HYPOTHESIS TESTING, EXPERIMENTS, AND PREDICTIONS

Knowledge is expanded when we can verify or falsify a hypothesis. That’s because the
experimental tests are constructed in such a way that the hypothesis is likely to be a
widely applicable explanation of certain facts, rather than an isolated case. This sort of
experiment is controlled, which means that the experimental setups differ by only one
variable (see Mill’s method of difference). The experimental group is the one that gets
the variable, while the control group does not.

Causal claims that result from experiments should reflect five criteria:

1) There should be a correlation between cause and effect.


2) The cause should precede the effect.
3) The cause should be in the proximity of the effect.
4) A set of necessary and sufficient conditions should exist.
5) Alternative explanations should be ruled out.

TOPIC 6: SCIENCE AND SUPERSTITION

One of the main features of scientific methodology is verification and falsifiability.


Recall from Chapter 4 that an appeal to ignorance is made when we infer from a lack of
evidence that something is or is not the case. Although there are times when a lack of
evidence should result in a judgment that the original claim is unsupported (as in a
criminal court), this is not so in scientific practices. If a hypothesis cannot be verified, it
should be retracted. In addition, a hypothesis must not be trivial. That is, it should not
be advanced in spite of everything we know to be true already.

98
ASSESSMENT

Name:__________________________________________ Date:_______________
Year & Section:___________________________________ Rating:______________

Direction: Encircle the letter of the correct answer.

Identify whether the intended causality in the following statements is a necessary


condition, a sufficient condition, or both a necessary and a sufficient condition.

1. Ingesting alcoholic beverages will cause one’s blood alcohol content to increase.

a. Sufficient condition
b. Necessary condition
c. Joint sufficient and necessary condition

2. On a 100-point grading scale, where a score of 90–100 points earns an “A”


grade, scoring 92 for a course earns an “A” grade.

a. Sufficient condition
b. Necessary condition
c. Joint sufficient and necessary condition

99
3. Dropping a crystal vase on a cement floor from five feet will cause the vase to
break.

a. Sufficient condition
b. Necessary condition
c. Joint sufficient and necessary condition

Determine which of Mill’s methods matches the description that follows.

4. Suppose all of the instances of an event have everything in common except one
item. Suppose also that another instance of the event also has everything in
common with the other instances, except one item. We can conclude that the item
that is not in common is causally connected to the event.

a. Method of agreement
b. Method of difference
c. Joint method of agreement and difference
d. Method of residues
e. Method of concomitant variations

5. If several of the instances of an event have only one item in common, then that
item is causally connected to the event.

a. Method of agreement
b. Method of difference
c. Joint method of agreement and difference
d. Method of residues
e. Method of concomitant variations

Identify the two most plausible hypotheses to explain the circumstances in the following
situation. In addition, for both hypotheses, identify the most plausible experiment that
would result in additional evidence that could be used to either confirm or refute the
hypotheses.

6. Helena doesn’t understand why she keeps earning substandard scores on her
logic exams. She’s taken two exams and can’t seem to score above a 72. Exam 3 is
coming up.

a. Hypothesis 1: Helena doesn’t have the right sort of mind for logic.
Hypothesis 2: Helena hasn’t been studying in a way conducive to success
in logic. Experiment: Helena adjusts her study habits in the days leading
up to her next exam.
b. Hypothesis 1: Helena hasn’t been studying in a way conducive to
success in logic.
Hypothesis 2: The last two logic exams have emphasized concepts and
skills Helena finds particularly difficult.
Experiment: Helena adjusts her study habits in the days leading up to her

100
next exam, focusing her attention on the particularly difficult concepts
and skills that will be tested.
c. Hypothesis 1: Helena doesn’t have the right sort of mind for logic.
Hypothesis 2: The professor gears the course toward math, computer
science, and philosophy majors.
Experiment: Helena should drop the class and enter one that better suits
her skill set.

7. Hector’s four horses, which normally eat their morning grains heartily, have
refused to eat this morning. They are eating their hay and drinking their water.

a. Hypothesis 1: The horses are colicky.


Hypothesis 2: The horses are not hungry.
Experiment: Hector should give the horses a muscle relaxant and walk
them until they pass manure. Then try feeding them the next morning,
according to their usual schedule.
b. Hypothesis 1: The horses are colicky.
Hypothesis 2: The horses are not hungry.
Experiment: Hector should call the vet to administer an enema, and then
resume their normal feeding schedule the next morning.
c. Hypothesis 1: There is something wrong with the grain.
Hypothesis 2: The feeding bins are contaminated.
Experiment: Hector should clean out the feeding bins, remove the grain
that’s been given, and try another serving, either of a different brand or
from another bag.

Determine the hypothesis, experiment, and prediction for the following fictional case
study.

8. Louelle ascribes her good health to lots of sleep, exercise, and a healthy diet. She
is one of a set of triplets. Her two siblings died more than 30 years ago, but Louelle
just celebrated her 97th birthday.

a. Hypothesis: Louelle’s good health is caused by lots of sleep and a


healthy diet.
Experiment: Eat healthy foods and get sufficient sleep.
Prediction: Live a long life.
b. Hypothesis: Live a long life.
Experiment: Louelle’s good health is caused by lots of sleep and a healthy
diet.
Prediction: Eat healthy foods and get sufficient sleep.
c. Hypothesis: Live a long life.
Experiment: Eat healthy foods and get sufficient sleep.
Prediction: Louelle’s good health is caused by lots of sleep and a healthy
diet.
d. Hypothesis: Louelle’s good health is caused by lots of sleep and a
healthy diet.
Experiment: Louelle’s good health is caused by lots of sleep and a healthy
diet.
Prediction: Eat healthy foods and get sufficient sleep.

101
Determine whether the study confirms or disconfirms the hypothesis, and what the
alternative explanation(s) may be.

9. Louelle ascribes her good health to lots of sleep, exercise, and a healthy diet. She
is one of a set of triplets. Her two siblings died more than 30 years ago, but Louelle
just celebrated her 97th birthday.

a. Confirm/Disconfirm: The evidence appears to confirm the


prediction.
Alternative Explanations: Louelle’s siblings could have died from causes
unrelated to their diet, exercise, and sleeping habits. In any case, there is
no prediction, just an explanation for a past event. As such, it does not
offer support for the hypothesis.
b. Confirm/Disconfirm: The evidence appears to confirm the
prediction.
Alternative Explanations: Louelle has good genes.
c. Confirm/Disconfirm: The evidence does not appear to confirm the
prediction.
Alternative Explanations: Louelle’s siblings could have died in accidents.
d. Confirm/Disconfirm: The evidence does not appear to confirm the
prediction.
Alternative Explanations: Louelle has been luckier than her siblings.

LESSON XV
ANALYZING ESSAY

TOPICS
1. Things to consider in analysing essay

LEARNING OUTCOMES
At the end of the lesson, student should be able to:
 Allow students to verify through evidence and facts from the topic of
interest.

TOPIC 1: THINGS TO CONSIDER IN ANALYSING ESSAY

 When analyzing a long essay, it’s often helpful to keep a set of notes next to you
which summarize the major points of each paragraph. When you look at the
entirety of your notes, you should be able to discern a thread of reasoning that
runs through the entire piece.
 It’s often easy to feel overwhelmed by a long essay. But consider reading
through it once without stopping, and then ask yourself what you remember as
being the most important ideas. It’s kind of like telling a friend about a movie
you just saw. You likely focus on a theme (“It’s a film about good people

102
triumphing over adversity,” or “It’s a film about the moral ambiguities of war.”)
or a summary of what happened (“So this guy comes home to find his wife has
left him. But then strange things happen to make him think she might have been
kidnapped. The rest of the story follows his search for the truth about what
happened to his wife.”)
 When analyzing a long essay, be sure to focus on the author’s reasoning. Doing
so will help you follow a coherent line of thought, rather than, say, random
observations that don’t contribute to the essay as a whole.
 When analyzing a long essay that involves abstract reasoning, try to come up
with concrete examples to help you better understand what the author means.

ASSESSMENT

Name:__________________________________________ Date:_______________
Year & Section:___________________________________ Rating:______________

Exercise Set 15. Encircle the letter of your choice.

Read the excerpt from Descartes’s Meditations on First Philosophy, and then answer the
questions that follow:
1. What does the meditator conclude is the case, given his new path? (Paragraph
12)
a. God really is deceiving him.
b. A malignant demon is using all of his powers to deceive the
meditator.
c. He is dreaming.
d. His senses can, in fact, be trusted.
2. How does the meditator propose to solve the problem of doubt? (Paragraph 10)
a. He does not propose to solve the problem, but resigns himself to a
lifetime of uncertainty.
b. He will not accept as true anything that can possibly be doubted.

103
c. He will examine all of his beliefs, one by one.
d. He will resort to faith in a divine force.
3. On what ground does the meditator reject his conditional trust in sensation?
(Paragraph 5)
a. He might be mentally ill.
b. He might be a figment of someone’s imagination.
c. He might be hallucinating.
d. He might be dreaming.
4. What does the meditator believe is doubtful? (Paragraph 3)
a. Thoughts
b. Emotions
c. Sensations
d. Himself
5. What does the meditator seek, even in his dream? (Paragraphs 6, 7, and 8)
a. Absolution
b. Confirmation that his senses are reliable
c. Certainty
d. Confidence

REFERENCES

BOOKS

Deborah J. Bennet (2004). Logic made easy : how to know when language deceives you
(1st ed). NY, USA: W.W. Norton & Company, Inc.

Merrilee H. Salmon (2013). Introduction to Logic and Critical Thinking, (6 th ed). Boston,
USA: Clark Baxter

Patrick J. Hurley, Lori Watson (2016). A Concise Introduction to Logic, (13th ed) Boston,
USA: Cengage Learning.

Stan Baronett (2018). Logic, (4th ed). USA: Oxford University Press.

INTERNET SOURCES

Barnes, Jonathan (2001). Early Greek Philosophy. 2nd ed. NY: Penguin Books.
Ferguson, Kitty (2008). The Music of Pythagoras. NY: Walker & Co.
July 28, 2020, https://public.wsu.edu/~delahoyd/mythology/pythagoras.html

Stan Baronett. (2012). Logic, 2nd edition, July 28, 2020,


https://global.oup.com/us/companion.websites/9780199846313/book/

104
The Florida State University. (2020). Department of Philosophy, July 28, 2020,
https://philosophy.fsu.edu/undergraduate-study/why-philosophy/What-is-Philosophy

105

You might also like